Bio Exam 1

Réussis tes devoirs et examens dès maintenant avec Quizwiz!

domain

(1)A taxonomic category above the kingdom level. The three domains are Archaea, Bacteria, and Eukarya. (2) A discrete structural and functional region of a protein.

Decide whether each of the following pairs of structures more likely represents analogy or homology, and explain your reasoning: (a) a porcupine's quills and a cactus's spines; (b) a cat's paw and a human's hand; (c) an owl's wing and a hornet's wing.

(a) Analogy, since porcupines and cacti are not closely related and since most other animals and plants do not have similar structures; (b) homology, since cats and humans are both mammals and have homologous forelimbs, of which the hand and paw are the lower part; (c) analogy, since owls and hornets are not closely related and since the structure of their wings is very different

SCIENTIFIC INQUIRY DRAW IT Mosquitoes resistant to the pesticide DDT first appeared in India in 1959, but now are found throughout the world. (a) Graph the data in the table below. (b) After examining the graph, hypothesize why the percentage of mosquitoes resistant to DDT rose rapidly. (c) Suggest an explanation for the global spread of DDT resistance. Month / 0 / 8 / 12 / Mosquitoes Resistant* to DDT / 4% / 45% / 77% / (*Mosquitoes were considered resistant if they were not killed within 1 hour of receiving a dose of 4% DDT.)

(a) Graph (b) The rapid rise in the percentage of mosquitoes resistant to DDT was most likely caused by natural selection in which mosquitoes resistant to DDT could survive and reproduce while other mosquitoes could not. (c) In India - where DDT resistance first appeared - natural selection would have caused the frequency of resistant mosquitoes to increase over time. If resistant mosquitoes then migrated from India (for ex, transported by wind, or in planes, ships, etc.) to other parts of the world, the frequency of DDT resistance would increase there as well.

Identify the theme or themes exemplified by (a) the sharp quills of a porcupine, (b) the development of a multicellular organism from a single fertilized egg, and (c) a hummingbird using sugar to power its flight.

(a) New properties emerge at successive levels of biological organizations: Structure and function are correlated. (b) Life's processes involve the expression and transmission of genetic information. (c) Life requires the transfer and transformation of energy and matter.

How does evolution account for (a) the similar mammalian forelimbs with different functions shown in Figure 19.16 and (b) the similar forms of the two distantly related mammals shown in Figure 19.18?

(a) despite their different functions, the forelimbs of different mammals are structurally similar because they all represent mods of a structure found in the common ancestor(b) convergent evolution: the similarities between the sugar glider and flying squirrel indicate that similar environments selected for similar adaptions despite different ancestry

WHAT IF? Consider two species that diverged while geographically separated but resumed contact before reproductive isolation was complete. Predict the outcome over time if the two species mated indiscriminately and (a) hybrid offspring survived and reproduced more poorly than offspring from intraspecific matings or (b) hybrid offspring survived and reproduced as well as offspring from intraspecific matings.

(a) reinforcement could occur. If it did, natural selection would cause prezygotic barriers to reproduction between parent species to strengthen over time, decreasing production of unfit hybrids, leading to completion of the speciation process. (b) indiscriminate mating between parent species would lead to the production of large numbers of hybrid offspring. As these hybrids mated with each other and with members of both parent species, gene pools of parent species could fuse over time, reversing speciation process.

Which is the correct taxonomic name for the leopard?

(in italics) Panthera pardus The correct use of binomial nomenclature requires both genus and species, both italicized if typed or underlined if it is hand-written. Genus is always written first and the first letter capitalized and species is always the second word and is lowercase.

taxon

(plural, taxa) A named taxonomic unit at any given level of classification.

SYNTHESIZE YOUR KNOWLEDGE This "kettle lake", formed 14,000 years ago when a glacier that covered the surrounding area melted. Initially devoid of animal life, over time the lake was colonized by invertebrates and other animals. Hypothesize how mutation, natural selection, genetic drift, and gene flow may have affected populations that colonized the lake.

- Regarding mutations effect, many organisms of the lake would have acquired favorable mutations, which would have helped them survive better in the lake - natural selection - when glacier would have melted organisms, which can survive cold climate and water would have been more common. Gradually temperature must have been rising, which would have lead to the selection of animals which can survive a certain level of heat - genetic drift - certain organisms, which would have migrated to this new lake, could have faced founders effect and bottleneck effect. This would have affected the allelic diversity of their characters - gene flow - if the lake was accessible to other ecosystems, then other populations of species colonizing the lake would have added allelic frequency of traits. This would have helped them in keeping larger gene pools. Higher genetic variability is always helpful

How did the frequencies of kdr genotypes change over time in A. coluzzii? Describe a hypothesis that accounts for these observations.

- The r allele was present in A. gambiae before 2006 and increased over time because individuals with the r allele are resistant to insecticides and pass their resistant genes to new generations.- The r allele was not present in A. coluzzii before 2006, but was present in A. coluzzii populations after hybrid individuals carrying the r allele were observed in 2006. This suggests that A. gambiae × A. coluzzii hybrids mated with A. coluzzii individuals, leading to the transfer of the adaptive allele r into A. coluzzii populations.

scientific evidence for evolution

- researchers have directly observed NS leading to adaptive evolution in many studies, including research on soapberry bug populations and on MRSA - organisms share characteristics because of common descent (homology) or because NS affects independently evolving species in similar environments in similar ways (convergent evolution) -fossils show that past organisms differed from living organisms, that many species have become extinct, and that species have evolved over long periods of time; fossils also document the origin of major new groups of organisms - evolutionary theory can explain biogeographic patterns

Consider a population in Hardy-Weinberg equilibrium for a character trait with these genotypic frequencies: AA = 0.25, Aa = 0.50 and aa = 0.25. If you remove all the homozygous dominants and allow the remaining population to reproduce (again under Hardy-Weinberg conditions), what will be the frequency of homozygous dominants in the next generation?

0.11 The new frequencies of the three genotypes are: AA = 0, Aa = 0.66 and aa = 0.33. If there were 100 individuals (carrying 200 alleles), 66/200 alleles would be A and p = 0.33. The frequency of homozygous dominant individuals would then be p2, or 0.11.

Use the observed genotype frequencies from the day 7 data to calculate the frequencies of the C^G allele (p) and the C^Y allele (q). (Remember that the frequency of an allele in a gene pool is the number of copies of that allele divided by the total number of copies of all alleles at that locus.)

0.484 C^G 0.516 C^Y

If a new species of plant is to be produced by means of allopolyploidy from two parental species that are 2n = 4 and 2n = 8, how many chromosomes would you expect in the somatic cells of the new species?

12 The two gametes would be n = 2 and n = 4. They would combine for a total of 6, which after doubling would be 12.

By applying a molecular clock, researchers have proposed that the first HIV-1 M invasion into humans occurred in the __________.

1930s This is the point in time when some scientists estimate that HIV jumped from simians to humans.

Calculate the value of the reproductive isolation index if (a) all of the matings within a population were successful, but none of the matings between populations were successful; (b) salamanders are equally successful in mating with members of their own population and members of another population.

2.0 0

In a certain group of African people, 4% are born with sickle-cell disease (homozygous recessive). If this group is in Hardy-Weinberg equilibrium, what percentage of the group is heterozygous and, thus, resistant to malaria?

32% The 4% born with sickle-cell disease are homozygous for the sickle hemoglobin allele, so q2 = 0.04 and q = 0.2. Because p = 1 -; q, p = 0.8. The heterozygotes would be equal to 2pq, or 0.32.

In Figure 22.8, approximately when did the most recent ancestor shared by Mammuthus (woolly mammoth), Asian elephants, and African elephants live?

5.5 MIL YRS

In a large population of bonobos, the frequency of the recessive allele is initially 0.1. What is the frequency of the dominant allele if there are two alleles of this gene?

90% p (the frequency of the dominant allele) = 1 -; q (the frequency of the recessive allele), or 0.9.

What does a branch point in a phylogenetic tree represent?

A branch point represents a point at which two evolutionary lineages split from a common ancestor. A branch point shows evolutionary relationships as a dichotomy.

Phylogenetic tree

A branching diagram that represents a hypothesis about the evolutionary history of a group of organisms

shared ancestral character

A character that is shared by members of a particular clade but that originated in an ancestor that is not a member of that clade.

polyploidy

A chromosomal alteration in which the organism possesses more than two complete chromosome sets. It is the result of an accident of cell division.

Which scenario would most likely result in the microevolution of a population of humans?

A colony of humans on the moon is isolated from Earth. Genetic drift (the founder effect) could occur.

binomial

A common term for the two-part, latinized format for naming a species, consisting of the genus and specific epithet; also called a binomen.

Is a dark brown mouse on dark-colored soil more likely to escape predation under a full moon or with no moon? What about a light brown mouse on light-colored soil? Explain.

A dark brown mouse is more likely to escape predation under a full moon on a dark-colored soil most likely because it is less visible to the eye of the owl. A light brown mouse on light-colored soil is also most likely to escape the owl's predation according to the data of the experiment conducted.

morphological species concept

A definition of species in terms of measurable anatomical criteria.

climate change

A directional change in temperature, precipitation, or other aspects of the global climate that lasts for three decades or more.

gene

A discrete unit of hereditary information consisting of a specific nucleotide sequence in DNA (or RNA, in some viruses).

variable

A factor that can change in an experiment

independent variable

A factor whose value is manipulated or changed during an experiment to reveal possible effects on another factor (the dependent variable).

dependent variable

A factor whose value is measured in an experiment to see whether it is influenced by changes in another factor ( the independent variable).

vestigial structure

A feature of an organism that is a historical remnant of a structure that served a function in the organism's ancestors

allopolyploid

A fertile individual that has more than two chromosome sets as a result of two different species interbreeding and combining their chromosomes.

hybrid zone

A geographic region in which members of different species meet and mate, producing at least some offspring of mixed ancestry

population

A group of individuals of the same species that live in the same area and interbreed, producing fertile offspring.

clade

A group of species that includes an ancestral species and all its descendants.

Describe some assumptions and limitations of molecular clocks.

A key assumption of molecular clocks is that nucleotide substitutions occur at fixed rates, and hence the number of nucleotide differences between two DNA seqences is proportional to the time since the sequences diverged from each other. Some limitations of molecular clocks: No gene marks time with complete precision; natural selection can favor certain DNA changes over others; nucleotide substitution rates can change over long periods of time (causing molecular clock estimates of when events in the distant past occurred to be highly uncertain); and the same gene can evolve at different rates in different organisms.

molecular clock

A method for estimating the time required for a given amount of evolutionary change, based on the observation that some regions of genomes evolve at constant rates.

What is a molecular clock? What assumption underlies the use of a molecular clock?

A molecular clock is a method of estimating the actual time of evolutionary events based on numbers of base changes in orthologous genes. It is based on the assumption that the regions of genomes being compared evolve at constant rates.

DNA (deoxyribonucleic acid)

A nucleic acid molecule, usually a double-stranded helix, in which each polynucleotide strand consists of nucleotide monomers with a deoxyribose sugar and the nitrogenous bases adenine (A), cytosine (C), guanine (G), and thymine (T); capable of being replicated and determining the inherited structure of a cell's proteins.

fossil

A preserved remnant or impression of an organism that lived in the past.

maximum parsimony

A principle that states that when considering multiple explanations for an observation, one should first investigate the simplest explanation that is consistent with the facts.

genetic drift

A process in which chance events cause unpredictable fluctuations in allele frequencies from one generation to the next. Effects of genetic drift are most pronounced in small populations.

natural selection

A process in which individuals that have certain inherited traits tend to survive and reproduce at higher rates than other individuals because of those traits.

adaptive evolution

A process in which traits that enhance survival or reproduction tend to increase in frequency in a population over time

Prezygotic barrier

A reproductive barrier that impedes mating between species or hinders fertilization if interspecific mating is attempted

postzygotic barrier

A reproductive barrier that prevents hybrid zygotes produced by two different species from developing into viable, fertile adults

strata

A rock layer formed when new layers of sediment cover older ones and compress them.

taxonomy

A scientific discipline concerned with naming and classifying the diverse forms of life.

systematics

A scientific discipline focused on classifying organisms and determining their evolutionary relationships.

experiment

A scientific test. Often carried out under controlled conditions that involve manipulating one factor in a system in order to see the effects of changing that factor.

organ

A specialized center of body function composed of several different types of tissues.

outgroup

A species or group of species from an evolutionary lineage that is known to have diverged before the lineage that contains the group of species being studied. An outgroup is selected so that its members are closely related to the group of species being studied, but not as closely related as any study-group members are to each other.

ingroup

A species or group of species whose evolutionary relationships are being examined in a given analysis.

genus

A taxonomic category above the species level, designated by the first word of a species' two-part scientific name.

kingdom

A taxonomic category, the second broadest after domain.

prokaryotic cell

A type of cell lacking a membrane-enclosed nucleus and membrane-enclosed organelles. Organisms with prokaryotic cells (bacteria and archaea) are called prokaryotes.

Eukaryotic cells

A type of cell that has a membrane-enclosed nucleus and other membrane-enclosed organelles. Organisms with eukaryotic cells (protists, plants, fungi, and animals) are called eukaryotes.

eukaryotic cell

A type of cell with a membrane-enclosed nucleus and membrane-enclosed organelles. Organisms with eukaryotic cells (protists, plants, fungi, and animals) are called eukaryotes.

inductive reasoning

A type of logic in which generalizations are based on a large number of specific observations.

deductive reasoning

A type of logic in which specific results are predicted from a general premise.

FOCUS ON A THEME: INFORMATION A typical prokaryotic cell has about 3,000 genes in its DNA, while a human cell has almost 21,000 genes. About 1,000 of these genes are present in both types of cells. (a) Based on your understanding of evolution, explain how such different organisms could have this same subset of 1,000 genes. (b) What functions might these shared genes have? Justify your choices.

A) All eukaryotic species came from a common ancestor. Millions of years ago, there were a meteorites fall that almost kills all living organisms on Earth, only a few survive and those evolve into all species as we know them now. And this is why into DNA code, you can find similar parts or sequences of genes in a lot of different species. Decent with modification explains life's unity and diversity. Unity in that all species descended from a common ancestor. Diversity in the modification/changes that evolved as species branched from their common ancestors. The 1000 genes that are the same represent the unity of life. The genes that are conserved between a prokaryote and a human cell are involved in core cell functions. Examples: Replication, transcription, translation, interactions with the environment, energy production, amino acid biosynthesis.

Bird guides once listed the myrtle warbler and Audubon's warbler as distinct species. Recently, these birds were reclassified as eastern and western forms of a single species, the yellow-rumped warbler. Which of the following pieces of evidence, if true, would be cause for this reclassification? A) The two forms interbreed often in nature, and their offspring survive and reproduce well. B) The two forms live in similar habitats and have similar food requirements. C) The two forms have many genes in common. D) The two forms are very similar in appearance.

A) The two forms interbreed often in nature, and their offspring survive and reproduce well.

In a comparison of birds and mammals, the condition of having four limbs is A) a shared ancestral character. B) a shared derived character. C) a character useful for distinguishing birds from mammals. D) an example of analogy rather than homology.

A) a shared ancestral character.

There are 25 individuals in population 1, all with genotype AA, and there are 40 individuals in population 2, all with genotype aa. Assume that these populations are located far from each other and that their environmental conditions are very similar. Based on the information given here, the observed genetic variation most likely resulted from A) genetic drift. B) gene flow. C) nonrandom mating. D) directional selection.

A) genetic drift.

DNA sequences in many human genes are very similar to the sequences of corresponding genes in chimpanzees. The most likely explanation for this result is that A) humans and chimpanzees share a relatively recent common ancestor. B) humans evolved from chimpanzees. C) chimpanzees evolved from humans. D) convergent evolution led to the DNA similarities.

A) humans and chimpanzees share a relatively recent common ancestor.

which of the following is not an observation or inference on which Darwin's theory of natural selection is based? A) poorly adapted individuals never produce offspring B) there is heritable variation among individuals C) because of overproduction of offspring, there is competition for limited resources D) a population can become adapted to its environment over time

A) poorly adapted individuals never produce offspring

Which of the following is an example of qualitative data? A) the fish swam in a zigzag motion B) the contents of the stomach are mixed ever 20 seconds C) the temperature decreased from 20 Celsius to 15 Celsius D) the six pairs of robins hatched an average of three chicks each

A) the fish swam in a zigzag motion

If you were using cladistics to build a phylogenetic tree of cats, which of the following would be the best outgroup? A) wolf B) domestic cat C) frog D) leopard

A) wolf

How many dark brown mice were caught in the light-colored soil enclosure on a moonlit night? How many dark brown mice were caught in the dark-colored soil enclosure on a moonlit night? On a moonlit night, would a dark brown mouse be more likely to escape predation by owls on dark- or light-colored soil? Explain your answer.

About 18 mice were caught by the owl. About 12 mice were caught by the owl. According to the data, a dark brown mouse would most likely be able to escape the owl's predation on dark-colored soil.

Combining the data from both graphs, estimate the number of mice caught in moonlight versus no-moonlight conditions. Which condition is optimal for predation by the owl? Explain.

About 73 mice were caught on a full moon versus about 94 mice caught on a moonless night. It is more optimal for the owl to prey on mice during a moonless night than a moonlit one.

A population of zooplankton is exposed to a small number of predatory fish that feed on the larger-sized (adult) zooplankton. Which prediction would most likely occur based on the principles of natural selection?

Adult zooplankton will start to reach sexual maturity when they are still relatively small. This would permit the zooplankton to reproduce in spite of the predation pressure on larger individuals.

(a) Which species concept(s) could you apply to both asexual and sexual species? (b) Which would be most useful for identifying species in the field? Explain.

All except the biological species concept can be applied to both asexual and sexual species because they define species on the basis of characteristics other than the ability to reproduce. In contrast, the biological species concept can be applied only to sexual species. (b) morphological species concept because based on appearance of organism

Explain the logic of using shared derived characters to infer phylogeny.

All features of organisms arose at some point in the history of life. In the group in which a new feature first arose, that feature is a shared derived character that is unique to that clade. The group in which each shared derived character first appeared can be determined, and the resulting nested pattern can be used to infer evolutionary history.

Which mechanism(s) can alter allele frequencies? - gene flow - natural selection - genetic drift - All of the choices above are correct.

All of the choices above are correct. Natural selection, gene flow, and genetic drift can all cause microevolution

In a hybrid zone, which would tend to lead to reinforcement? - reduced hybrid fertility - hybrid breakdown - sexual selection - reduced hybrid viability - All of the choices are correct.

All of the choices are correct. All of the choices can reinforce the reproductive barriers between populations.

During his voyage around the world, Darwin was inspired to think about evolution by __________. - fossils he collected - the works of others such as Lamarck - studying adaptations of organisms to their environments - the unique organisms he saw in the Gálapagos Islands - All of the choices are correct.

All of the choices are correct. During his voyage on the HMS Beagle, Darwin was inspired to think about evolution by the writings of Lamarck, fossils, the adaptations of organisms such as the Galápagos finches, and the uniqueness of organisms.

The two-part format of the scientific name, referred to as binomial, ensures that __________. - each species is assigned a unique name - systematists can easily specify the closest relatives of any species - each species has a name that is understandable regardless of language barriers among scientists - All of the choices are correct.

All of the choices are correct. Scientific names provide many benefits for clarity in understanding connections among organisms.

During periods of rapid environmental change, what may happen to a species that was well-suited to the former environment? - Traits that were favorable in the original environment may be detrimental in the new environment. - Individuals with particular traits that provide an advantage in the new environment will have higher reproductive success. - The population may change so much in adapting to the new environment that it is considered a new species. - The species may go extinct. - All of the choices are correct.

All of the choices are correct. When environments change, species will either adapt, possibly giving rise to new species, or go extinct.

Which can be considered a biological system? - A pond - A single live cell - A salmon's cardiovascular system - The biosphere - All of the choices are correct.

All the choices are correct A system is a combination of components that function together. It can be defined at any scale.

Which is true when talking about scientific theories? - It is general enough to spin off many new, testable hypothesis - It is supported by a much greater body of evidence - It is much broader in scope than a hypothesis - None of the choices are correct when explaining theories in science. - All the choices are correct except the fourth.

All the choices are correct except the fourth. All of the statements can support an accepted theory in science.

ecosystem

All the organisms in a given area as well as the abiotic factors with which they interact; one or more communities and the physical environment around them.

community

All the organisms that inhabit a particular area; an assemblage of populations of different species living close enough together for potential interaction.

WHAT IF? Is allopatric speciation more likely to occur on an island close to a mainland or on a more isolated island of the same size? Explain your prediction.

Allopatric speciation would be less likely to occur on an island near a mainland than on a more isolated island of the same size. We expect this result because continued gene flow between mainland populations and those on a nearby island reduces the chance that enough genetic divergence will take place for allopatric speciation to occur.

Explain why natural selection is the only evolutionary mechanism that consistently leads to adaptive evolution in a population.

Although both gene flow and genetic drift can increase the frequency of advantageous alleles in a population, they can also decrease the frequency of advantageous alleles or increase the frequency of harmful alleles. Only natural selection consistently results in an increase in the frequency of alleles that enhance survival or reproduction. Thus, natural selection is the only mechanism that consistently leads to adaptive evolution.

FOCUS ON A THEME: EVOLUTION Using at least TWO examples, explain how the process of evolution is revealed by the imperfections of living organisms.

Although natural selection can improve the match between organisms and their environments, the evolutionary process can also lead to imperfections in organisms. A central reason for this is that evolution does not design organisms from scratch to match their environments and ways of life by works instead by a process of decent what with modifications, organisms inherit a basic forms from their ancestors, and that form is modified by naturals selection over time, as a result, such as a bat has wings that are not perfectly designs, but rather represent modifications of forelimbs that bat ancestors used for walking. Imperfections in organisms result from a variety of other constraints, such as lack of genetic variation for the trait in question, and the fact that adaptations often represent compromises (since organisms must do many different things, and a perfect design for one activity might impair the performance of another activity).

How is a mailing address analogous to biology's hierarchical classification system?

An address pinpoints a location by tracking from broader to narrower categories—a state, city, zip, street, and building number. This is analogous to the groups-subordinate-to-groups structure of biological taxonomy.

systems biology

An approach to studying biology that aims to model the dynamic behavior of whole biological systems based on a study of the interactions among the system's parts.

cladistics

An approach to systematics in which organisms are placed into groups called clades based primarily on common descent.

Explain how the following statement is inaccurate: "Antibiotics have created drug resistance in MRSA."

An environmental factor such as a drug does not create new traits, such as drug resistance, but rather selects for traits among those that are already present.

shared derived character

An evolutionary novelty that is unique to a particular clade.

speciation

An evolutionary process in which one species splits into two or more species.

controlled experiement

An experiment designed to compare an experimental group with a control group; ideally, the two groups differ only in the factor being tested.

theory

An explanation that is broader in scope than a hypothesis, generates new hypotheses, and is supported by a large body of evidence.

organisms

An individual living thing, consisting of one or more cells.

autopolyploid

An individual that has more than two chromosome sets that are all derived from a single species

tissue

An integrated group of cells with a common function, structure, or both.

How could natural selection have led to the evolution of adaptations such as camouflaging coat color in beach mice?

Ancestors of the beach mouse may have exhibited variations in their coat color. Because of the prevalence of visual predators, the better camouflaged (lighter) mice may have survived longer and been able to produce more offspring. Over time, a higher and higher proportion of individuals in the population would have had the adaptation of lighter fur that acted to camouflage the mouse.

The three domains you learned about in Concept 1.2 can be represented in the tree of life as the three main branches, with three subbranches on the eukaryotic branch being the kingdoms Plantae, Fungi, and Animalia. What if fungi and animals are more closely related to each other than either of these kingdoms is to plants--as recent evidence strongly suggests? Draw a simple branching pattern that symbolizes the proposed relationship between these three eukaryotic kingdoms.

Ancestral Eukaryotes: -plants -fungi and animals

organelles

Any of several kinds of membrane-enclosed structures with specialized functions, suspended in the cytosol of eukaryotic cells.

Which domain is prokaryotic?

Archaea Archaea are among the simplest kinds of organisms, the prokaryotes.

The scala naturae, or scale of nature, is based on the ideas of __________.

Aristotle Aristotle believed that all living forms could be arranged on a ladder, or scale, of increasing complexity. Each form of life, perfect and permanent, had its allotted rung on this ladder.

MAKE CONNECTIONS Review the relationship between genotype and phenotype (see Figure 11.6). Suppose that in a particular pea population, flowers with the white phenotype are favored by natural selection. Predict what would happen over time to the frequency of the p allele in the population, and explain your reasoning.

As long as the white phenotype (encoded by the genotype pp) continues to be favored by natural selection, the frequency of the p allele will likely increase over time in the population. If the proportion of white individuals increases relative to purple individuals, the frequency of the recessive p allele will also increase relative to that of the P allele, which only appears in purple individuals (some of which also carry a p allele).

VISUAL SKILLS In Figure 20.4, which similarly inclusive taxon is represented as descending from the same common ancestor as Canidae? A) Felidae B) Mustelidae C) Carnivora D) Lutra

B) Mustelidae

The upper forelimbs of humans and bats have fairly similar skeletal structures, whereas the corresponding bones in whales have very different shapes and proportions. However, genetic data suggest that all three kinds of organisms diverged from a common ancestor at about the same time. Which of the following is the most likely explanation for these data? A) Forelimb evolution was adaptive in people and bats, but not in whales. B) Natural selection in an aquatic environment resulted in significant changes to whale forelimb anatomy. C) Genes mutate faster in whales than in humans or bats. D) Whales are not properly classified as mammals.

B) Natural selection in an aquatic environment resulted in significant changes to whale forelimb anatomy.

Which of the following is NOT an observation or inference on which natural selection is based? A) There is heritable variation among individuals. B) Poorly adapted individuals never produce offspring. C) Species produce more offspring than the environment can support. D) Only a fraction of an individual's offspring may survive.

B) Poorly adapted individuals never produce offspring.

All the organisms on your campus make up A) an ecosystem. B) a community. C) a population. D) a taxonomic domain.

B) a community

If the nucleotide variability of a locus equals 0%, what are the gene variability and number of alleles at that locus? A) gene variability=0%; number of alleles=0 B) gene variability=0%; number of alleles=1 C) gene variability=0%; number of alleles=2 D) gene variability>0%; number of alleles=2

B) gene variability=0%; number of alleles=1

The largest unit within which gene flow can readily occur is a A) population. B) species. C) genus. D) hybrid.

B) species.

A controlled experiment is one that A) proceeds slowly enough that a scientist can make careful records of the results. B) tests experimental and control groups in parallel. C) is repeated many times to make sure the results are accurate. D) keeps all variables constant.

B) tests experimental and control groups in parallel.

Identify the following elements of hypothesis-based science in this example: (a) question, (b) hypothesis, (c) prediction, (d) control group, and (e) experimental group.

Based in the experiment with the guppies, it can be concluded that natural selection is a factor of evolution. What is a hypothesis? A hypothesis is an explanation put forward to explain an observation without prior testing. From the observations in the guppies; The question is whether natural selection is favors coloration in guppies The hypothesis is that natural selection determines coloration in guppies The prediction is that guppies with bright coloration will thrive in a predator free environment The control group is the source pool the experimental group is the transfer pool the conclusion from the data is that guppies with bright coloration are naturally selected Returning the guppies from the transfer pool to the source pool will essentially reduce the population of the bright colored guppies due to predation. Therefore, it can be seen from the guppies that natural selection drives evolution of species.

In the deserts of the southwestern United States, the soils are mostly sandy, with occasional large regions of black rock derived from lava flows that occurred 1.7 million years ago. Mice are found in both sandy and rocky areas, and owls are known predators. What might you expect about coat color in these two mouse populations? Explain. How would you use this ecosystem to further test the camouflage hypothesis?

Based on the mouse coloration in Figure 1.25, you might expect that the mice that live on the sandy soil would be lighter in color and those that live on the lava rock would be much darker. And in fact, that is what researchers have found. You would predict that each color of mouse would be less preyed upon in its native habitat than it would be in the other habitat. You could repeat the Hoekstra experiment with colored models, painted to resemble these two types of mouse. Or you could try transplanting some of each population to its non-native habitat than it would be in the other habitat. In the live mouse transplantation experience experiment, you would have to do controls to eliminate the variable represented by the transplanted mice being in a new, unknown territory. You could control for the transplantation process by transplanting some dark mice from one are of lava rock to one far distant, and some light mice from one area of sandy soil to a distant area.

Explain why phylogenies based on different genes can yield different branching patterns for the tree of all life.

Because of horizontal gene transfer, some genes in eukaryotes are more closely related to bacteria, while others are more closely related to archaea; thus, depending on which genes are used, phylogenetic trees constructed from DNA data can yield conflicting results.

The frequency of allele a is 0.45 for a population in Hardy-Weinberg equilibrium. What are the expected frequencies of genotypes AA, Aa, and aa?

Because the frequency of allele a is 0.45,the frequency of allele A must be 0.55. Thus, the expected genotype frequencies are p2 = 0.3025 for genotype AA, 2pq = 0.495 for genotype Aa, and q2 = 0.2025 for genotype aa.

WHAT IF? Predict what might happen if gene flow were re-established at step 3 in this process.

Because the populations had only just begun to diverge, it is likely that any existing barriers to reproduction would weaken over time.

SCIENTIFIC INQUIRY DRAW IT In this chapter, you read that bread wheat (Triticum aestivum) is an allohexaploid, containing two sets of chromosomes from each of three different parent species. Genetic analysis suggests that the three species pictured following this question each contributed chromosome sets to T. aestivum. (The capital letters here represent sets of chromosomes, each of which can be traced to a particular species, not individual genes.) Evidence also indicates that the first polyploidy event was a spontaneous hybridization of the early cultivated wheat species T. monococcum and a wild Triticum grass species. Draw a diagram of one possible chain of events that could have produced the allohexaploid T. aestivum. (pg 458)

Bread wheat possesses the scientific name T. aestivum. It resides under the grouping of the most significant crops in the world. Its domestication corresponds with the commencement of farming, and ever since, it has been continually under the choice of human beings. Step 2: Einkorn Einkorn is another name for the species of wheat Triticum monococcum. These species fall under the category of diploid ancestral wheat. They exhibit an association with durum and bread wheat. Step 3: Diagram that indicates the spontaneous hybridization of the early cultivated wheat species T. monococcum and a wild Triticum grass species The ancestral species of wheat are described as T. monococcum, Wild Triticum, and wild T. tauschii. So, first, the fusion of species T. monococcum and Wild Triticumoccurs. This results in the formation of AB which is sterile species. Afterward, both species (T. monococcum and Wild Triticum) fuse with the third species, which is wild T. tauschii. This results in the formation of ABD, indicated as T. aestivum. It is also called bread wheat. This is again a sterile species.

A fruit fly population has a gene with two alleles, A1 and A2. Tests show that 70% of the gametes produced in the population contain the A1 allele. If the population is in Hardy-Weinberg equilibrium, what proportion of the flies carry both A1 and A2? A) 0.7 B) 0.49 C) 0.42 D) 0.21

C) 0.42

Plant species A has a diploid number of 12. Plant species B has a diploid number of 16. A new species, C, arises as an allopolyploid from A and B. The diploid number for species C would probably be A) 14. B) 16. C) 28. D) 56.

C) 28.

Which of the following best demonstrates the unity among all organisms? A) emergent properties B) descent with modification C) DNA structure and function D) natural selection

C) DNA structure and function

Which of the following statements best distinguishes hypotheses from theories in science? A) Theories are hypotheses that have been proved. B) Hypotheses are guesses; theories are correct answers. C) Hypotheses usually are relatively narrow in scope; theories have broad explanatory power. D) Theories are proved true; hypotheses are often contradicted by experimental results.

C) Hypotheses usually are relatively narrow in scope; theories have broad explanatory power.

Within six months of effectively using methicillin to treat S. aureus infections in a community, all new S. aureus infections were caused by MRSA. How can this best be explained? A) A patient must have become infected with MRSA from another community. B) In response to the drug, S. aureus began making drug-resistant versions of the protein targeted by the drug. C) Some drug-resistant bacteria were present at the start of treatment, and natural selection increased their frequency. D) S. aureus evolved to resist vaccines.

C) Some drug-resistant bacteria were present at the start of treatment, and natural selection increased their frequency.

Males of different species of the fruit fly Drosophila that live in the same parts of the Hawaiian Islands have different elaborate courtship rituals. These rituals involve fighting other males and making stylized movements that attract females. What type of reproductive isolation does this represent? A) habitat isolation B) temporal isolation C) behavioral isolation D) gametic isolation

C) behavioral isolation

Protists and bacteria are grouped into different domains because A) protists eat bacteria B) bacteria are not made of cells C) protists have a membrane-bound nucleus D) protists are photosynthetic

C) protists have a membrane-bound nucleus

Systems biology is mainly an attempt to A) analyze genomes from different species B) simplify complex problems by reducing the system into smaller, less complex units. C) understand the behavior of entire biological systems by studying interactions among its component parts. D) build high-throughput machines for the rapid acquisition of biological data

C) understand the behavior of entire biological systems by studying interactions among its component parts.

How does natural selection fashion organisms?

Chance and the environment interact with natural selection, so that the best available traits are selected for. The ideal traits for a particular environment may not exist or may have been removed from the population by chance events. Natural selection can only act on traits that exist in the population.

FOCUS ON A THEME: INTERACTIONS Write a short essay (about 100-150 words) evaluating whether changes to an organism's physical environment are likely to result in evolutionary change. Use an example to support your reasoning.

Change in an organism's physical environment will cause the organism to adapt to fit in the surrounding environment. This process is called as adaptation. This process causes the evolution of the organism into a new species. The environment affects the evolution of the species , because change in environment put pressure on the species to adapt in change environmental conditions. Natural selection brings the change in the organism as the organism best suited for the environment survives.The example of evolution is the giraffe neck. Giraffe evolved by the development of tall neck slowly by increasing their neck length, so that they can eat leaf from the tall plants.

Label the part of the diagram that represents the most recent common ancestor of frogs and humans

Circle the branch point splitting the frog lineage from the lineage leading to lizards, chimps, and humans.

Why is natural selection called a theory?

Compared to a hypothesis, a scientific theory is usually more general and substantiated by a much greater amount of evidence. Natural selection is an explanatory idea that applies to all kinds of organisms and is supported by vast amounts of evidence of various kinds.

How might secondary sex characteristics in males differ from those in females in a species in which females compete for mates?

Compared to males, it is likely that the females of such species would be larger, more colorful, endowed with more elaborate ornamentation (for example, a large morphological feature such as the peacock's tail), and more apt to engage in behaviors intended to attract mates or prevent other members of their sex from obtaining mates.

Two garden plots were planted with corn. The soil was similar in each, and equal amounts of water were applied to each plot. One plot was fertilized, and the other was not. The experimenters measured the yield as bushels of corn from each plot. The plot that did not receive the fertilizer was the __________.

Control plot The control refers to the plot that does not receive the experimental treatment or the variable being tested (in this case, fertilizer).

VISUAL SKILLS In this tree diagram, what is the sister taxon of the clade that includes dinosaurs and their most recent common ancestor? Explain. I-----------Lizards and snakes -I I---------Crocodilians I---I I---Ornithischian dinosaurs I---I I-Saurischian dinosaurs I---I I------Birds

Crocodilians are the sister taxon to the dinosaur clade (which includes birds) because crocodilians and the dinosaur clade share an immediate common ancestor that is not shared by any other group.

WHAT IF? Why did the researchers split each female frog's eggs into two batches for fertilization by different males? Why didn't they mate each female with a single male frog? (pg 437)

Crossing a single female's eggs with both an SC and an LC male's sperm allowed the researchers to directly compare the effects of the males' contribution to the next generation since both batches of offspring had the same maternal contribution. This isolation of the male's impact enabled researchers to draw conclusions about differences in genetic "quality" between the SC and LC males.

MAKE CONNECTIONS Compare Figure 10.11 with Figure 22.19. What cellular process could cause the hybrid chromosomes in Figure 22.19 to contain DNA from both parent species? Explain.

Crossing over

The modern idea of extinction as a common occurrence in Earth's history was first proposed in the early 19th century writings of __________.

Cuvier Cuvier advocated catastrophism, speculating that extinctions apparent in the fossil record corresponded to a catastrophe, such as a flood or drought.

Which of the following factors would not contribute to allopatric speciation? A) A population becomes geographically isolated from the parent population. B) The separated population is small, and genetic drift occurs. C) The isolated population is exposed to different selection pressures than the ancestral population. D) Gene flow between the two populations is extensive.

D) Gene flow between the two populations is extensive.

Which of the following best describes the logic of scientific inquiry? A) If I generate a testable hypothesis, tests and observations will support it. B) If my prediction is correct, it will lead to a testable hypothesis. C) If my observations are accurate, they will support my hypothesis. D) If my prediction is correct, my hypothesis is supported.

D) If my prediction is correct, my hypothesis is supported. (I can expect certain test results)

VISUAL SKILLS Based on the tree below, which statement is NOT correct? A) Goats and humans form a sister group. B) Salamanders are a sister group to the group containing lizards, goats, and humans. C) Salamanders are as closely related to goats as to humans. D) Lizards are more closely related to salamanders than to humans. I-------------Salamander ----I I--------Lizard I----I I---Goat I----I I---Human

D) Lizards are more closely related to salamanders than to humans.

Which of the following observations helped Darwin shape his concept of descent with modification? A) Species diversity declines farther from the equator. B) Fewer species live on islands than on the nearest continents. C) Birds live on islands located farther from the mainland than the birds' maximum nonstop flight distance. D) South American temperate plants are more similar to South American tropical plants than to European temperate plants.

D) South American temperate plants are more similar to South American tropical plants than to European temperate plants.

Three living species X, Y, and Z share a common ancestor T, as do extinct species U and V. A grouping that consists of species T, X, Y, and Z (but not U or V) makes up A) a monophyletic taxon. B) an ingroup, with species U as the outgroup. C) a polyphyletic group. D) a paraphyletic group.

D) a paraphyletic group.

To apply parsimony to constructing a phylogenetic tree, A) choose the tree that assumes all evolutionary changes are equally probable. B) choose the tree in which the branch points are based on as many shared derived characters as possible. C) choose the tree that represents the fewest evolutionary changes, in either DNA sequences or morphology. D) choose the tree with the fewest branch points.

D) choose the tree with the fewest branch points.

Natural selection changes allele frequencies because some __________ survive and reproduce better than others. A) alleles B) loci C) species D) individuals

D) individuals

According to the punctuated equilibria model, A) natural selection is unimportant as a mechanism of evolution. B) given enough time, most existing species will branch gradually into new species. C) most evolution occurs in sympatric populations. D) most new species accumulate their unique features relatively rapidly as they come into existence, then change little for the rest of their duration as a species.

D) most new species accumulate their unique features relatively rapidly as they come into existence, then change little for the rest of their duration as a species.

Sparrows with average-sized wings survive severe storms better than those with longer or shorter wings, illustrating A) the bottleneck effect. B) disruptive selection. C) frequency-dependent selection. D) stabilizing selection.

D) stabilizing selection.

The relative lengths of the frog and mouse branches in the phylogenetic tree in Figure 20.13 indicate that A) frogs evolved before mice. B) mice evolved before frogs. C) the homolog evolved more rapidly in the mouse lineage. D) the homolog evolved more slowly in the mouse lineage.

D) the homolog evolved more slowly in the mouse lineage.

What is the molecule that can account for both the unity and the diversity of life?

DNA The uniformity of coding in DNA is virtually universal. Life's variety arises from variations in the nucleotide sequences of inherited DNA molecules, the substance of genes.

What combination of independent variables led to the highest predation level in enclosures with light-colored soil? What combination of independent variables led to the highest predation level in enclosures with dark-colored soil?

Dark colored mice on a moonless night. Light colored mice on a moonlit night. the opposite conditions for each are most deadly for each colored mice.

FOCUS ON A THEME: EVOLUTION Darwin suggested looking at a species' close relatives to learn what its ancestors may have been like. Explain how his suggestion anticipates recent methods, such as phylogenetic bracketing and the use of outgroups in cladistic analysis.

Darwin asserted that it was possible to recognize ancestral characteristics of a species if we look at how close relatives are. This statement resembles the concept used in the process called phylogenetic bracketing (because this method looks for characteristics in individuals by analyzing their position in a phylogenetic tree and comparing characteristics with species in similar positions) and resembles the process called cladistic analysis (which states that the classification of organisms must be made according to their evolutionary relationships) Both Darwin's speech, phylogenetic bracketing, and cladistic analysis are approaches that are based on solid evidence for how groups of living species are related through common ancestry between these groups.

Which observation and inference led Charles Darwin to his theory of natural selection as a mechanism for evolution? - Individuals with heritable traits best-suited to the local environment will generally produce a disproportionate number of healthy, fertile offspring. - Individuals in a population of any species vary in many heritable traits. - Individuals of a population are unequal in the likelihood of surviving and reproducing. - A population of any species has the potential to produce far more offspring than will survive to produce offspring of their own. - Darwin synthesized his theory of natural selection from all of the listed observations and inferences.

Darwin synthesized his theory of natural selection from all of the listed observations and inferences. All of the listed observations and inferences led Charles Darwin to his theory of natural selection as the mechanism for evolution.

biological species concept

Definition of a species as a group of populations whose members have the potential to interbreed in nature and produce viable, fertile offspring, but do not produce viable, fertile offspring with members of other such groups.

ecological species concept

Definition of a species in terms of ecological niche, the sum of how members of the species interact with the nonliving and living parts of their environment.

evolution

Descent with modification; the process by which species accumulate differences from their ancestors as they adapt to different environments over time; also defined as a change in the genetic composition of a population from generation to generation.

rooted

Describing a phylogenetic tree that contains a branch point (often, the one farthest to the left) representing the most recent common ancestor of all taxa in the tree.

genetic variation

Differences among individuals in the composition of their genes or other DNA sequences

A population has 700 individuals, 85 of genotype AA, 320 of genotype Aa, and 295 of genotype aa. What are the frequencies of alleles A and a?

Each individual has two alleles, so the total number of alleles is 1,400. To calcu- late the frequency of allele A, note that each of the 85 individuals of genotype AA has two A alleles, each of the 320 individuals of genotype Aa has one A allele, and each of the 295 individuals of genotype aa has zero A alleles. Thus, the frequency (p) of allele A is p =((2 85) + (1 320) + (0 * 295))/1,400 = 0.35There are only two alleles (A and a) in our population, so the frequency of allele a must be q = 1 - p = 0.65.

MAKE CONNECTIONS Explain how the origin of mitochondria would have led to extensive horizontal gene transfer. (See Figure 4.16.)

Eukaryotes are hypothesized to have originated when a heterotrophic prokaryote (an archaeal host cell) engulfed a bacterium that would later become an organelle found in all eukaryotes—the mitochondrion. Over time, a fusion of organisms occurred as the archaeal host cell and its bacterial endosymbiont evolved to become a single organism. As a result, we would expect the cell of a eukaryote to include both archaeal DNA and bacterial DNA, making the origin of eukaryotes an example of horizontal gene transfer.

In terms of populations, how do we define evolution?

Evolution is a change in a population's allele frequencies over generations. This is the definition of microevolution, which focuses on evolutionary change in populations.

FOCUS ON A THEME: INFORMATION In a short essay (100-150 words), explain how genetic information—along with an understanding of the process of descent with modification—enables scientists to reconstruct phylogenies that extend hundreds of millions of years back in time.

Evolutionary biology illustrates both the pattern and processes. The processes of evolution are natural selection and other mechanisms, which modifies the genetic structure of the populations. These processes result in evolutionary patterns, that is, the products generated by evolution with time. Phylogeny refers to the evolutionary history of a species or a group of species. In order to redevelop phylogeny, the scientists use systematics, that is, an analytical method to categorize the diversity and finding the evolutionary associations between the extinct and the living species. The evidence used to redevelop phylogenies can be attained from the fossil record and from the biochemical, morphological, and genetic similarities between the species. The scientists are functioning to develop a universal tree of all life, which will get refined with the gathering of new information.

microevolution

Evolutionary change below the species level; change in the allele frequencies in a population over generations.

Starting with the molecular level in Figure 1.3, write a sentence that includes components from the previous (lower) level of biological organization, for example, "A molecule consists of atoms bonded together." Continue with organelles, moving up the biological hierarchy.

Examples: A molecule consists of atoms bonded together. Each organelle has an orderly arrangement of molecules. Photosynthetic plant cells contain organelles called chloroplasts. A tissue consists of a group of similar cells. Organs such as the heart are constructed from several tissues. A complex multicellular organism, such as a plant, has several types of organs, such as leaves and roots. A population is a set of organisms of the same species. A community consists of populations of the various species inhabiting a specific area. An ecosystem consists of a biological community along with the nonliving factors important to life, such as air, soil, and water. The biosphere is made up of all of Earth's ecosystems.

In the Lake Victoria cichlids, what appears to be contributing to fusion of different species?

Females have a difficult time visually selecting males of the same species in the murky, polluted water. Researchers are finding that sexual selection, which had been reinforcing reproductive barriers between species, is less effective in the murky water of the present-day lake.

Thinking about the muscles and nerves in your hand, how does the activity of text messaging reflect the four unifying themes of biology described in this section?

Finger movements rely on the coordination of the many structural components of the hand (muscles, nerves, bless, etc.) each of which is composed of elements from lower levels of biological organization (cells, molecules). The development of the hand relies on the genetic information encoded in chromosomes found in cells throughout the body. To power the finger movements that result in a text message, muscle and nerve cells require chemical energy that they transform in powering muscle contraction or in propagating nerve impulses. Finally, all of the anatomical and physiological features that allow the activity of texting are the outcome of a process of natural selection that resulted in the evolution of hands and of the mental facilities for use of language.

I---Fishes ----I I---Frogs I----I I---Lizards I----I I---Chimps I----I I---Humans How many sister taxa are shown

Four: (4)chimps-humans, (3)lizards-chimps/humans; (2)frogs-lizards/chimps/humans (1)fishes-frogs/lizards/chimps/humans

I---Fishes I---Frogs I---Lizards I---Chimps I---Humans According to this tree, which group or groups of organisms are most closely related to frogs?

Frogs are most closely related to a group consisting of lizards, chimps, and humans in this tree

(a) Calculate the kdr genotype frequencies in A. gambiae for each time period. To do this, divide the number of individuals that have a given genotype by the total number of individuals observed for that time period. (b) How did the kdr genotype frequencies change over time? Describe a hypothesis that accounts for these observations.

From pre-2006 to post-2006, the frequency of the r/r genotype increased dramatically while the frequencies of +/+ and +/r genotypes decreased dramatically. From pre-2006 to post-2006, the frequencies of the r/r and +/r genotypes increased dramatically while the frequency of +/+ decreased dramatically.

Eukaryotic organisms that decompose dead organisms and absorb the nutrients are generally found in which kingdom?

Fungi These are eukaryotic decomposers that obtain their nutrients by breaking down dead organisms and organic wastes.

What is the first thing that must happen in order for speciation to occur?

Gene flow between populations must be interrupted. Although several other factors come into play, as long as there is gene flow between populations, speciation is extremely unlikely.

Universal phylogenetic trees built from different genes sometimes give inconsistent results. What assumption, on which phylogenetic trees are based, has probably been violated during the history of life, causing this confusion?

Genes are passed vertically from one generation to the next. Phylogenetic trees assume vertical gene transfer, but many organisms also use horizontal gene transfer. Many scientists postulate that horizontal gene transfer was far more prevalent during the early history of life.

Why was the five-kingdom system abandoned for a three-domain system?

Genetic data indicated that many prokaryotes differed as much from each other as they did from eukaryotes. This indicated that organisms should be grouped into three "super kingdoms' or domains (Archaea, Bacteria, Eukarya). These data also indicated that the previous kingdom Monera (which had contained all the prokaryotes) did not make biological sense and should be abandoned. Later genetic and morphological data also indicated that the former kingdom Protista (which had primarily contained single-celled organisms) should be abandoned because some protists are more closely related to plants, fungi, or animals than they are to other protists.

Distinguish genetic drift from gene flow in terms of (a) how they occur and (b) their implications for future genetic variation in a population.

Genetic drift results from chance events that cause allele frequencies to fluctuate at random from generation to generation; within a population, this process tends to decrease genetic variation over time. Gene flow is the transfer of alleles between populations, a process that can introduce new alleles to a population and hence may increase its genetic variation (albeit slightly, since rates of gene flow are often low).

founder effect

Genetic drift that occurs when a few individuals become isolated from a larger population and form a new population whose gene pool composition is not reflective of that of the original population.

bottleneck effect

Genetic drift that occurs when the size of a population is reduced, as by a natural disaster or human actions. Typically, the surviving population is no longer genetically representative of the original population.

Explain why genetic variation within a population is a prerequisite for evolution.

Genetic variation is an important force in evolution as it allows natural selection to increase or decrease frequency of alleles already in the population.

MAKE CONNECTIONS If a population stopped reproducing sexually (but still reproduced asexually), how would its genetic variation be affected over time? Explain. (See Concept 10.4.)

Genetic variation would most likely drop over time. In sexual reproduction is where alleles are shuffled and create variations.

heterozygote advantage

Greater reproductive success of heterozygous individuals compared to homozygotes; tends to preserve variation in a gene pool.

sister taxa

Groups of organisms that share an immediate common ancestor and hence are each other's closest relatives.

analogous

Having characteristics that are similar because of convergent evolution, not homology.

Which is a true statement about Charles Darwin? - He proposed natural selection as the mechanism of evolution. - He was the first to conclude that Earth is billions of years old. - He was the first to discover that living things can change, or evolve. - He based his theory on the inheritance of acquired characteristics. - He worked out the principles of population genetics.

He proposed natural selection as the mechanism of evolution. The proposal of a testable mechanism was a key point in the acceptance of his theory.

Tay-Sachs disease, which is lethal, results from being homozygous recessive for one gene. Which statement is true?

Heterozygous individuals will survive and may pass the recessive allele on to their offspring. The diploid nature of most eukaryotes hides a considerable amount of genetic variation from selection, in the form of recessive alleles in heterozygotes.

DRAW IT In (b), circle the most inclusive clade for which a hinged jaw is a shared ancestral character. (pg 409 in book)

Hinged jaws are a shared ancestral character for the group that includes frogs, turtles, and leopards. Thus, you should have circled the frog, turtle, and leopard lineages, along with their most recent common ancestor.

Why is it necessary to distinguish homology from analogy to infer phylogeny?

Homologous characters result from shared ancestry. As organisms diverge over time, some of their homologous characters will also diverge. The homologous characters of organisms that diverged long ago typically differ more than do the homologous characters of organisms that diverged more recently. As a result, differences in homologous characters can be used to infer phylogeny. In contrast, analogous characters result from convergent evolution, not shared ancestry, and hence can give misleading estimates of phylogeny.

FOCUS ON A THEME: EVOLUTION Explain the biological basis for assigning all human populations to a single species. Can you think of a scenario by which a second human species could originate in the future?

Humans are grouped under one species because of the ability for all humans to mate with each other. There are no differences throughout humans that allows for them to not mate. Also because of our similar biological make-ups we are all considered humans. Possibly in the future if an event occurs where there are two groups of people that cannot mate with each other there will be a second human species, but I have no idea how that could happen.

What are hybrid zones, and why can they be viewed as "natural laboratories" in which to study speciation?

Hybrid zones are regions in which members of different species meet and mate, producing some offspring of mixed ancestry. Can be viewed as natural laboratory because scientists can directly observe factors that cause reproductive isolation.

MAKE CONNECTIONS Review the process of meiosis in Figure 10.8. Describe how an error during meiosis could lead to polyploidy.

If all homologs failed to separate during anaphase I of meiosis, some gametes would end up with extra set of chromosomes. If gamete with extra set of chromosomes fused with normal gamete, triploid would result; if two gametes with extra set of chromosomes fused with each other, tetraploid would result.

What factors can support the long-term stability of a hybrid zone if the parent species live in different environments?

If hybrids are selected against, the hybrid zone could persist if individuals from parent species regularly travel into the zone and mate. If they aren't selected against, there is no cost to the continued production of hybrids, and large numbers of hybrid offspring may be produced. However, natural selection may keep the gene pools of the 2 parent species distinct, thus preventing the loss of the parent species causing the hybrid zone to be stable over time.

Which statement is most clearly derived inductively? - If the animals observed require organic molecules as nutrients, then it can be concluded that all animals require organic molecules as nutrients. - If all flying animals are birds, then it can be concluded that bats are birds. - An elephant is warm-blooded because it is a mammal. - Because worms lack bones, they are classified as invertebrates. - A paramecium moves by means of the rhythmic motion of its cilia.

If the animals observed require organic molecules as nutrients, then it can be concluded that all animals require organic molecules as nutrients. Induction moves from a set of specific observations (humans require organic molecules, fish require organic molecules, etc.) to reach a general conclusion (all animals require organic molecules).

WHAT IF? Why were all flies used in the mating preference tests reared on a standard medium (rather than on starch or maltose)? (pg 446)

If this had not been done, the strong preference of "starch flies" and "maltose flies" to mate with like-adapted flies could have occurred simply because the flies could detect (for example, by sense of smell) what they potential mates had eaten as larvae - and they preferred to mate with the flies that had a similar smell to their own

phyla

In Linnaean classification, the taxonomic category above class.

family

In Linnaean classification, the taxonomic category above genus.

order

In Linnaean classification, the taxonomic category above the level of family.

classes

In Linnaean classification, the taxonomic category above the level of order.

basal taxon

In a specified group of organisms, a taxon whose evolutionary lineage diverged early in the history of the group.

Contrast allopatric and sympatric speciation. Which type of speciation is more common, and why?

In allopatric speciation , a new species forms while in geographic isolation from its parent; In sympatric speciation a new species forms in absence of geographic isolation, geographic isolation greatly reduces the gene flow of populations, allopatric is more common.

FOCUS ON A THEME: ORGANIZATION Heterozygotes at the sickle-cell locus produce both normal and abnormal (sickle-cell) hemoglobin (see Concept 11.4). When hemoglobin molecules are packed into a heterozygote's red blood cells, some cells receive relatively large quantities of abnormal hemoglobin, making these cells prone to sickling. In a short essay (approximately 100-150 words), explain how these molecular and cellular events lead to emergent properties at the individual and population levels of biological organization.

In an individual's molecular and cellular level, the heterozygote alleles are preferred over the homozygote alleles. In humans, the heterozygote alleles are found at a locus of beta polypeptide subunit of hemoglobin, while on the other hand, the homozygous alleles found at a similar locus are prone to sickle cell disease. The individuals carrying homozygous alleles exhibit sickle-shaped RBCs and they also possess low oxygen-carrying capacity, which ultimately results in brain, kidney, or heart failure. However, in the case of heterozygous alleles, the configurations of RBCs are of two kinds, that is, normal shaped and sickle-shaped. Thus, there are not enough sickle-shaped cells to result in the condition. The heterozygote alleles are resistant to malaria, thus, in tropical areas, where malaria is a prime issue the heterozygote alleles are preferred over the homozygote dominant alleles as they are vulnerable to the infection and over the homozygote recessive alleles who has sickle cell disease.

WHAT IF? Suppose that female cichlids living in the murky waters of a polluted lake could not distinguish colors well. In such waters, how might the gene pools of these species change over time? (pg 449)

In murky waters where females distinguish colors poorly, females of each species might often with mate with males of other species.

punctuated equilibria

In the fossil record, long periods of apparent stasis, in which a species undergoes little ot no morphological change, interrupted by relatively brief periods of sudden change.

Imagine that part of a population of South American finches is blown by a storm onto an island far offshore and manages to survive and reproduce there for a period of 10,000 years. After that period, a climate change results in lower sea levels and the reconnection of the island with the mainland. Members of the formerly isolated island finch population can now interact freely with members of the original mainland population. Which observation would, by itself, lead you to conclude unequivocally that the island finch population had evolved into a distinct species, according to the biological species concept?

Individuals from the different populations sometimes mate with each other, but all of the resulting eggs are sterile. The biological species concept defines a species as a population whose members interbreed successfully to produce viable, fertile offspring but who cannot produce viable, fertile offspring with members of other species.

Contrast inductive reasoning with deductive reasoning.

Inductive reasoning derives generalizations from specific cases; deductive reasoning predicts specific outcomes from general premises.

What are the roles of gathering and interpreting data in scientific inquiry?

Inductive reasoning is used in forming hypotheses, while deductive reasoning leads to predictions that are used to test hypotheses.

adaptation

Inherited characteristic of an organism that enhances its survival and reproduction in a specific environment.

The idea of using molecules as clocks to time evolutionary events is very attractive, but there are many problems in applying the technique. What is the best way to get reliable results?

It is important to use as many genes as possible. With this approach, fluctuations in evolutionary rate will tend to average out. It is almost impossible to find a gene that has been unaffected by natural selection during its entire existence. Using many genes has been shown to give dates that match the fossil record fairly closely.

To which domain is the domain Eukarya most closely related?

It is not clear whether eukaryotes are more closely related to bacteria or archaea. Studies of rRNA genes indicate that eukaryotes and archaea are more closely related, but genes that influence metabolism indicate a closer relationship between eukaryotes and bacteria. Horizontal gene transfer has likely been at work here.

Would two small, geographically isolated populations in very different environments be likely to evolve in similar ways? Explain.

It is unlikely that two such populations would evolve in similar ways. Since their environments are very different, the alleles favored by natural selection would probably differ between the two populations. Although genetic drift may have important effects in each of these small populations, drift causes unpredictable changes in allele frequencies, so it is unlikely that drift would cause the populations to evolve in similar ways. Both populations are geographically isolated, suggesting that little gene flow would occur between them (again making it less likely that they would evolve in similar ways).

Comparison of human fossils with living humans seems to show that there have been no significant physical changes in Homo sapiens in 30,000 to 50,000 years. What might an advocate of punctuated equilibrium say about this?

Lack of change is consistent with the punctuated equilibrium model. In punctuated equilibrium, new species change little after budding from their parent species.

cell

Life's fundamental unit of structure and function; the smallest unite of organization that can perform all activities required for life.

SYNTHESIZE YOUR KNOWLEDGE This West Indian manatee (Trichechus manatus) is an aquatic mammal. Like amphibians and reptiles, mammals are tetrapods (vertebrates with four limbs). Explain why manatees are considered tetrapods even though they lack hind limbs, and suggest traits that manatees likely share with leopards and other mammals (see Figure 20.12b). Discuss how early members of the manatee lineage might have differed from today's manatees.

Manatees are considered tetrapods because their skeletons display the remains of a pelvis, indicating that they evolved from a tetrapod ancestor and are therefore in the same clade as other tetrapods. They likely share various derived characteristics with other mammals (eg hair, live birth, etc). as well as ancestral traits (eg pentadactyl limbs). Early members of the manatee lineage probably originally had 4 legs, and as time progressed the hind legs were lost. (The term tetrapod refers to evolutionary lineage and not to the actual phenotype of the organism.)

Which is a key observation that must be explained in a unifying theory about life?

Many basic characteristics are shared by all living things. Organisms do share many basic characteristics. Darwin reasoned that the diverse organisms found on Earth today evolved from a few common ancestors.

INTERPRET THE DATA For 1981, 1987, and 1990, compare the frequency of left-mouthed individuals among breeding adults to the frequency of left-mouthed individuals in the entire population. What do the data indicate about when natural selection favors left-mouthed individuals over right-mouthed individuals (or vice versa)? Explain.

Most of the breeding adults had the opposite phenotype of that which was most common in the population.

What qualitative observation led to the quantitative study outlined in Figure 1.21?

Mouse coat color matches the environment for both beach and inland populations.

Typically, most of the nucleotide variability that occurs within a genetic locus does not affect the phenotype. Explain why.

Much of the nucleotide variability at a genetic locus occurs within introns. Nucleotide variation at these sites typically does not affect the phenotype because introns do not code for the protein product of the gene. (Note: In certain circumstances, it is possible that a change in an intron could affect RNA splicing and ultimately have some phenotypic effect on the organism, but such mechanisms are not covered in this introductory text.) There are also many variable nucleotide sites within exons. However, most of the variable sites within exons reflect changes to the DNA sequence that do not change the sequence of amino acids encoded by the gene (and hence may not affect the phenotype)

__________ and __________ generate variation, whereas __________ results in an adaptation to the environment.

Mutation, sexual recombination, natural selection Mutation is the original source of variation, and sexual recombination causes new combinations of existing alleles. Natural selection results in favorable characteristics being passed on to the next generation.

The gene that causes sickle-cell disease is present in a higher percentage of residents of sub-saharan Africa than among those of African descent living in the United States. Discuss an evolutionary process that could account for the different percentages among residents of the two regions.

Natural selection could be operating. Malaria is present in sub-Saharan Africa, so there might be an advantage to people with the sickle-cell disease form of the gene that makes them more able to survive and pass on their genes to offspring. Among those of African descent living in the United States, where malaria is absent, there would be no advantage, so they would be selected against more strongly, resulting in fewer individuals with the sickle-cell disease form of the gene.

directional selection

Natural selection in which individuals at one end of the phenotypic range survive or reproduce more successfully than do other individuals.

disruptive selection

Natural selection in which individuals on both extremes of a phenotypic range survive or reproduce more successfully than do individuals with intermediate phenotypes.

stabilizing selection

Natural selection in which intermediate phenotypes survive or reproduce more successfully than do extreme phenotypes.

In what sense is natural selection more "predictable" than genetic drift?

Natural selection is more "predictable" in that it alters allele frequencies in a nonrandom way: It tends to increase the frequency of alleles that increase the organism's reproductive success in its environment and decrease the frequency of alleles that decrease the organism's reproductive success. Alleles subject to genetic drift increase or decrease in frequency by chance alone, whether or not they are advantageous.

balancing selection

Natural selection that maintains two or more phenotypic forms in a population

What is the importance of neutral variation in evolution?

Neutral variation increases genetic variation, allowing a population to carry more alleles that may help it respond to environmental change. A trait that is neutral in one environment may be advantageous in another. If a trait confers an advantage (or disadvantage), natural selection can act on it, and evolution may occur.

emergent properties

New properties that arise with each step upward in the hierarchy of life, owing to the arrangement and interactions of parts as complexity increases.

Thinking about your answers to question 5, provide a simple statement describing conditions that are especially deadly for either color of mouse.

No moon and on dark colored soil

Is it circular reasoning to calculate p and q from observed genotype frequencies and then use those values of p and q to test whether the population is in Hardy-Weinberg equilibrium? Explain your answer. (Hint: Consider a specific case, such as a population with 195 individuals of genotype AA, 10 of genotype Aa, and 195 of genotype aa.)

No, this is not an example of circular reasoning. Calculating p and q from observed genotype frequencies does not imply that those genotype frequencies must be in Hardy-Weinberg equilibrium. For example, consider a population that has 195 individuals of genotype AA, 10 of genotype Aa, and 195 of genotype aa. Calculating p and q from these values yields p=q=0.5p=q=0.5. Using the Hardy-Weinberg equation, the predicted equilibrium frequencies are p2=0.25p2=0.25 for genotype AA, 2pq=0.52pq=0.5 for genotype Aa, and q2=0.25q2=0.25 for genotype aa. Since there are 400 individuals in the population, these predicted genotype frequencies indicate that there should be 100 AA individuals, 200 Aa individuals, and 100 aa individuals—numbers that differ greatly from the values that we used to calculate p and q.

SCIENCE, TECHNOLOGY, AND SOCIETY In the United States, the rare red wolf (Canis rufus) has been known to hybridize with coyotes (Canis latrans), which are much more numerous. Although red wolves and coyotes differ in terms of morphology, DNA, and behavior, genetic evidence suggests that living red wolf individuals are actually hybrids. Red wolves are designated as an endangered species and hence receive legal protection under the Endangered Species Act. Some people think that their endangered status should be withdrawn because the remaining red wolves are hybrids, not members of a "pure" species. Do you agree? Why or why not?

No. The status of endangered species should not be withdrawn. I do not agree. If they were taken off and all of the hybrids would go extinct there would be no more red wolves at all. The chances of hybrids to still be existent are not possible because the one thing that still is carrying the alleles of the red wolves would be gone.

To distinguish a particular clade of mammals within the larger clade that corresponds to class Mammalia, would hair be a useful character? Why or why not?

No; hair is a shared ancestral character common to all mammals and thus is not helpful in distinguishing different mammalian subgroups

Which species concept defines a species as a set of organisms with a unique genetic history? - biological species concept - sexual species concept - ecological species concept - morphological species concept - None of the choices is correct.

None of the choices is correct. Biological species concept - This is the classical definition that defines a species as a population or group of populations whose members have the potential to interbreed with one another in nature and to produce viable, fertile offspring, but cannot produce viable, fertile offspring with members of other populations. Sexual species concept - This is not a defined species concept. ecological species concept - This concept views a species in terms of its ecological niche, its role in a biological community. morphological species concept - This concept is used to characterize a species in terms of body shape, size, and other distinct structural features.

Describe how over-reproduction and heritable variation relate to evolution by natural selection.

Observations: 1) individuals in a pop vary in their heritable characteristics 2) organisms produce more offspring than the environment can support CAUSE: Inferences: 1) individuals that are well suited to their environment tend to leave more offspring than other individuals 2) over time, favorable traits accumulate in the pop

Lake Victoria, in Africa, is home to a group of related fishes known as cichlids. Many of these fishes are similar in appearance but have different feeding habits. What is the best method for scientists to determine conclusively whether the fish are members of a population that has a lot of variation or members of entirely different species?

Observe the fish in their natural environment for possible instances of interbreeding leading to several generations of fertile offspring. The biological species concept defines a species as a population whose members interbreed successfully to produce viable, fertile offspring but who cannot produce viable, fertile offspring with members of other species.

hybrid

Offspring that results from the mating of individuals from two different species or from two true-breeding varieties of the same species.

Under which conditions would a dark brown mouse be most likely to escape predation at night? A light brown mouse?

On a moonlit night on dark colored soil. No moon light on light-colored soil.

Bacteria

One of two prokaryotic domains, the other being Archaea.

Archaea

One of two prokaryotic domains, the other being Bacteria.

Of all the mutations that occur in a population, why do only a small fraction become widespread?

Only a small fraction of mutations are beneficial and increase the fitness of a species (ability to survive the selection practice).

What insight did Darwin gain from reading Thomas Malthus's essay on human suffering?

Organisms have the capacity to over-reproduce. Malthus contended that human suffering is a product of humans reproducing faster than food and other resources can increase. This is actually a characteristic of many life-forms on Earth, though Malthus's work conflicts with economic facts as described by Adam Smith.

How does the concept of descent with modification explain both the unity and diversity of life?

Organisms share characteristics (the unity of life) because they share common ancestors; the great diversity of life occurs because new species have repeatedly formed when descendant organisms gradually adapted to different environments, becoming different from their ancestors.

After invoking selective breeding of domesticated species as evidence that groups of organisms are capable of change, Darwin then proposed that natural populations can change as well. On which two lines of evidence did he base this proposal?

Organisms within a population vary, and all populations produce more offspring than can be supported by the environment, resulting in competition for survival within the population. Darwin used these two lines of evidence to support his theory of descent with modification in natural populations.

WHAT IF? For each theme discussed in this section, give an example not mentioned in the text.

Organization (Emergent Properties): The ability of a human heart to pump blood requires an intact heart; it is not a capability of any of the heart's tissues or cells working alone. Organization (Structure and Function): The strong, sharp teeth of a wolf are well suited to grasping and dismembering its prey. Information: Human eye color determined by the combination of genes inherited from the two parents. Energy and Matter: A plant, such as stored fuel. Animals can eat parts of the plant and use the food for energy to carry out their activities. Interactions (Ecosystems): A mouse eats food, such as nuts or grasses, and deposits some of the food material as wastes (feces and urine). Construction of a nest rearranges the physical environment and may hasten degradation of some of its components. The mouse may also act as food for a predator. Interactions (Molecules): When your stomach is full, it signals your brain to decrease your appetite. Evolution: All plants have chloroplasts, indicating their descent from a common ancestor.

When reconstructing phylogenies, is it more useful to compare orthologous or paralogous genes? Explain.

Orthologous genes should be used; for such genes the homology results from speciation and hence reflects evolutionary history.

monophyletic

Pertaining to a group of taxa that consists of a common ancestor and all of its descendants. A monophyletic taxon is equivalent to a clade.

paraphyletic

Pertaining to a group of taxa that consists of a common ancestor and some, but not all, of its descendants.

polyphyletic

Pertaining to a group of taxa that includes distantly related organisms but does not include their most recent common ancestor

These caterpillars of the moth Nemoria arizonaria owe their different appearances to chemicals in their diets, not to differences in their genotypes. (a) Caterpillars raised on a diet of oak flowers resemble the flowers, whereas (b) their siblings raised on oak leaves resemble oak twigs.

Phenotypic variation Some variation in a population is from the environment influencing phenotype; Nonheritable variation. The main source of genetic variation arises from sexual reproduction shuffling parental alleles during meiosis and fertilization.

Rabbits and guinea pigs both belong to Class Mammalia. This means they must also both belong to __________.

Phylum Chordata Class is less inclusive than phylum; organisms that are in the same class are also in the same phylum.

Natural selection tends to act at which of the following levels?

Population Natural selection selects for organisms within a population that are best adapted to their local environment.

What is the term for a group of individuals from one species living within a defined area?

Population Only the organisms of a given species that have the opportunity to interact with one another are considered to be within the same population

What conclusion do you draw from the data presented above?

Predation results in selection for more drab coloration in pike-cichlid pools

WHAT IF? Suppose two bird species live in a forest and are not known to interbreed. One species feeds and mates in the treetops and the other on the ground. But in captivity, the birds can interbreed and produce viable, fertile offspring. What type of reproductive barrier most likely keeps these species separate in nature? Explain.

Prezygotic barrier because live in similar environments and can breed successfully in captivity and different preferences in habitats could result in habitat isolation.

Which kingdom within the domain Eukarya is composed of organisms that are generally single-celled?

Protista The eukaryotic protists are usually single-celled or unicellular organisms.

data

Recorded observations

endemic

Referring to a species that is confined to a specific geographic area

How does science differ from technology?

Science aims to understand natural phenomena and how they work, while technology involves application of scientific discoveries for a particular purpose or to solve a specific problem.

Though natural selection leads to adaptation, nature abounds with examples of organisms that are less than ideally suited for their lifestyles. There are several reasons why.

Selection can act only on existing variations. Natural selection favors only the fittest phenotypes among those currently in the population, which may not be the ideal traits. New advantageous alleles do not arise on demand. Evolution is limited by historical constraints. Each species has a legacy of descent with modification from ancestral forms. Evolution does not scrap the ancestral anatomy and build each new complex structure from scratch; rather, evolution co-opts existing structures and adapts them to new situations. We could imagine that if a terrestrial animal were to adapt to an environment in which flight would be advantageous, it might be best just to grow an extra pair of limbs that would serve as wings. However, evolution does not work this way; instead, it operates on the traits an organism already has. Thus, in birds and bats, an existing pair of limbs took on new functions for flight as these organisms evolved from nonflying ancestors. Adaptations are often compromises. Each organism must do many different things. A seal spends part of its time on rocks; it could probably walk better if it had legs instead of flippers, but then it would not swim nearly as well. We humans owe much of our versatility and athleticism to our prehensile hands and flexible limbs, but these also make us prone to sprains, torn ligaments, and dislocations: Structural reinforcement has been compromised for agility. Organisms face many such trade-offs in which the ability to perform one function may reduce the ability to perform another—and as with seals and humans, those trade-offs can restrict adaptive evolution. Chance, natural selection, and the environment interact. Chance events can affect the subsequent evolutionary history of populations. For instance, when a storm blows insects or birds hundreds of kilometers over an ocean to an island, the wind does not necessarily transport those individuals that are best suited to the new environment. Thus, not all alleles present in the founding population's gene pool are better suited to the new environment than the alleles that are "left behind." In addition, the environment at a particular location may change unpredictably from year to year, again limiting the extent to which adaptive evolution results in organisms being well suited to current environmental conditions. ^^With these four constraints, evolution does not result in perfect organisms. Natural selection operates on a "better than" basis. We can, in fact, see evidence for evolution in the many imperfections of the organisms it produces.

frequency-dependent selection

Selection in which the fitness of a phenotype depends on how common the phenotype is in a population

WHAT IF? Suppose two plant populations exchange pollen and seeds. In one population, individuals of genotype AA are most common (9,000 AA, 900 Aa, 100 aa), while the opposite is true in the other population (100 AA, 900 Aa, 9,000 aa). If neither allele has a selective advantage, what will happen over time to the allele and genotype frequencies of these populations?

Selection is not important at this locus; furthermore, the populations are not small, and hence the effects of genetic drift should not be pronounced. Gene flow is occurring via the movement of pollen and seeds. Thus, allele and genotype frequencies in these populations should become more similar over time as a result of gene flow.

FOCUS ON A THEME: INFORMATION In sexually reproducing species, each individual inherits DNA from both parents. In a short essay (100-150 words), apply this idea to what occurs when organisms of two species that have homologous chromosomes mate and produce (F1) hybrid offspring. What percentage of the DNA in the F1 hybrids' chromosomes comes from each parent species? As the hybrids mate and produce F2 and later-generation hybrid offspring, describe how recombination and natural selection may affect whether the DNA in hybrid chromosomes is derived from one parent species or the other.

Sexual reproduction is an early evolutionary modernization after the appearance of eukaryotic cells. During sexual reproduction, the genetic material of two individuals is put together to produce genetically-diverse offspring that is not the same from their parents. Actually there are more eukaryotes reproduce sexually is the proof of its evolutionary success. Most of the animals, it is actually the only way of reproduction. The genetic diversity of sexually-produced offspring is thought to produced species a better chance of surviving in a changing or unpredictable environment. The variation that sexual reproduction make among offspring is very important to reproduction of the population and the survival . In sexual reproduction, different mutations are continually reshuffled from one generation to the next when different parents combine their unique genomes; this results in an increase of genetic diversity. On average, a sexually-reproducing population will leave more offspring than an otherwise similar asexually-reproducing population.

Which statement is true about sexual selection?

Sexual selection can result in sexual dimorphism—marked differences between the sexes in secondary sexual characteristics that are not associated directly with reproduction. Sexual dimorphism is a result of sexual selection.

analogy

Similarity between two species that is due to convergent evolution rather than to descent from a common ancestor with the same trait.

homology

Similarity in characteristics resulting from a shared ancestry.

FOCUS ON A THEME: EVOLUTION In a short essay (100-150 words), describe Darwin's view of how natural selection resulted in both unity and diversity of life on Earth. Include in your discussion some of his evidence. (For help in writing good essays, see "Writing Tips and Rubrics" in the Study Area of Mastering Biology under "Additional Resources.")

So the unity that we see in life on Earth stems really from to fax one. Because all life on Earth is descended from a common ancestor and therefore, no matter how diverse life hasn't can become we all rose from the same organisms so many years ago. So even though we're on our own paths for a Sfar Aziz, we can see in the distance we always came from the same starting point, the same common ancestor. So that's one reason there's unity on Earth. And in Darwin's do he saw that he saw the fact that the finches all seem to have a common ancestor even though they diverse. He saw that as well. Um, secondly, we see unity in life on Earth because all life is connected in the complex ecosystems that we see in nature. Um, natural selection works through the interactions that organisms have with each other, and therefore no one species could succeed or change without another species there to push it along. And we see that in predator prey relationships, we see that the better that eagles get seeing mice, the better than my skin at hiding from eagles and the better that their camouflage becomes and the better, and vice versa. And the eagles get better at spotting and seeing, and then maybe they'll branch in different organisms and stuff in. So the unity that comes from that is the fact that every animal, every organism, is influencing the other organisms just as much. And they have to be interwoven in order to succeed. And okay, so now moving on to the diversity of life on Earth Um, there the diversity that we see results from the fact that natural selection is constantly pushing species along and in different directions. Organisms even within the same species often find themselves going down different paths of becoming completely new species of their own. Simply adding to the diversity of life that we see on Earth. Natural selection is this constant force that works to branch the tree of life in never ending directions. And Darwin saw this as well in his famous famous, um, Galapagos finches that he saw the birds with their different beaks nor the different kinds of foods. He saw that all these finches had taken different directions, even though they may have seemed similar. One time they went down their own paths and they have it. Very broad, big themes of biology and including evolution. So evolution is typically thought of in this very Darwinian sense, which is what this question gets at. And the idea is that evolution is driven by this concept of survival of the fittest, and this is where natural selection comes in. So if you have a trait that is beneficial, then you're more likely to survive more likely to pass on that trait and then the future. Generations of organisms are more likely tohave that trait, and this leads to two main themes. Diversity and unity and quick clams. They almost seem to contradict each other, but not not at least evolutionarily speaking. So one thing we can think of is Darwin's finches right there, the birds and they have different beaks based on the different conditions. So maybe one beak was really good for seeds, and then you had to beat. That was really good for nuts. And you have another beak that's really good for worms or something. And so, depending on how much rain father was, depending on which of these finches, it was more prevalent at different times and in different islands. So that's how we get diversity. There's also this underlying sense of commonality, right? All these birds are very similar genetically minus the beaks. And so you could even think about, um, leave it like bone structure. Okay, well, so the bones, the leg and front leg bones or leg and arm bones of, say, humans and horses and cats are all very similar. Okay, so they're similar among many mammals. Um, despite they're being other sense of diversity of species. Right? So even within this diversity, there is a lot of similarities, even at the genomic level. Um, you may have heard something on the lines of how we share 90 plus percent of our genome with this animal of that animal rights. So there is this, because diversity stems from some common ancestral organism. Um, we share an ancestor and therefore a lot of characteristics with many other species. So we get then both diversity and and unity, and so just gotta summarize that idea into the 100 ish words shouldn't be too hard

How did some strains of Staphylococcus aureus become resistant to antibiotic drugs?

Some members of the bacteria population must have had a genetic variation that made them resistant to antibiotics. Those bacteria that survived the various antibiotic drugs passed on to subsequent generations the genes that enabled them to resist the drug.

WHAT IF? Suppose that two species, A and B, have similar appearances but very divergent gene sequences, while species B and C have very different appearances but similar gene sequences. Which pair of species is more likely to be closely related: A and B, or B and C? Explain.

Species B and C are more likely to be closely related.

FOCUS ON A THEME: EVOLUTION Explain why anatomical and molecular features often fit a similar nested pattern. In addition, describe a process that can cause this not to be the case.

Specific anatomical and molecular features are shared by a narrow range of species are not shared by a more broad range of species. However, other features are shared not only among their narrow range of species but also a broader range. The process of convergent evolution producing analogous structures may confuse this.

homologous structures

Structures in different species that are similar because of common ancestry.

First, make sure you understand how the graphs are set up. Graph A shows data from the light-colored soil enclosure and graph B from the dark-colored enclosure, but in all other respects the graphs are the same. There is more than one independent variable in these graphs. What are the independent variables, the variables that were tested by the researcher? Which axis of the graphs has the independent variables? What is the dependent variable, the response to the variables being tested? Which axis of the graphs has the dependent variable?

The X-axis has the independent variable. The dependent variable is the number of mice caught by the owl. The dependent variable is located on the Y-axis.

gene pool

The aggregate of all copies of every type of allele at all loci in every individual in a population. The term is also used in a more restricted sense as the aggregate of alleles for just one or a few loci in a population.

technology

The application of scientific knowledge for practical purposes, often involving industry or commerce but also including uses in basic research.

What is true of parsimonious trees?

The best tree requires the fewest evolutionary events to have occurred in the form of shared derived characters. This statement is true regarding trees based on morphological characters. For phylograms based on DNA, the most parsimonious tree requires the smallest number of base changes in DNA.

relative fitness

The contribution an individual makes to the gene pool of the next generation, relative to the contributions of other individuals in the population.

Eukarya

The domain that includes all eukaryotic organisms.

biosphere

The entire portion of Earth inhabited by life; the sum of all the planet's ecosystems.

proteome

The entire set of proteins expressed by a given cell, tissue, or organism.

convergent evolution

The evolution of similar features in independent evolutionary lineages

phylogeny

The evolutionary history of a species or group of related species

reproductive isolation

The existence of biological factors (barriers) that impede members of two species from producing viable, fertile offspring.

Next, use the Hardy-Weinberg equation (p2+2pq+q2=1) to calculate the day 7 expected frequencies of genotypes C^GC^G,C^GC^Y, and C^YC^Y for a population in Hardy-Weinberg equilibrium.

The expected frequency of genotype CGCG is p 2 = 0.484 × 0.484 = 0.234.The expected frequency of genotype CGCY is 2pq = 2 × 0.484 × 0.516 = 0.499.The expected frequency of genotype CYCY is q 2 = 0.516 × 0.516 = 0.266.

Humans and chimpanzees are sister species. Explain what this statement means.

The fact that humans and chimpanzees are sister species indicates that we share a more recent common ancestor with chimpanzees than we do with any other living primate species. But that does not mean that humans evolved from chimpanzees or vice versa, instead it indicate that both umans and chimpanzees are descendants of that common ancestor.

allopatric speciation

The formation of new species in populations that are geographically isolated from one another.

sympatric speciation

The formation of new species in populations that live in the same geographic area

SCIENTIFIC INQUIRY INTERPRET THE DATA For populations of the marine mussel Mytilus edulis around Long Island, New York, researchers measured the frequency of a particular allele (lap94) for an enzyme involved in regulating the mussel's saltwater balance. They presented their data as a series of pie charts linked to sampling sites within Long Island Sound, where the salinity is highly variable, and along the coast of the open ocean, where salinity is constant (see the table that follows). (a) Create a data table for the 11 sampling sites by estimating the frequency of (lap94) from the pie charts. (Hint: Think of each pie chart as a clock face to help you estimate the proportion of the shaded area.) (b) Graph the frequencies for sites 1-8 to show how the frequency of this allele changes with increasing salinity in Long Island Sound (from southwest to northeast). Evaluate how the data from sites 9-11 compare with the data from the sites within the Sound. (c) Considering the various mechanisms that can alter allele frequency, construct a hypothesis that explains the patterns you observe in the data and that accounts for the following observations: (1) The (lap94) allele helps mussels maintain osmotic balance in water with a high salt concentration but is costly to use in less salty water; and (2) mussels produce larvae that can disperse long distances before they settle on rocks and grow into adults. (pg 439)

The frequency of the lap94 allele is higher at sites 9−11 than it is within the Sound; however, the northeast edge of the Sound has nearly the same frequency of the lap94 allele as do sites 9−11.The frequency of the lap94 allele is higher at sites 9−11 (open ocean) than it is at sites 1−7 (within the Sound). The northeast edge of the Sound (site 8) has about the same frequency of the lap94 allele as do the open ocean sites. The frequency of the lap94 allele at different sites results from an interaction between selection and gene flow.In the southwest portion of the Sound, salinity is relatively low, and selection against the lap94 allele is strong. Moving toward the northeast and into the open ocean, where salinity is relatively high, selection favors a high frequency of the lap94 allele. However, because mussel larvae disperse long distances, gene flow prevents the lap94 allele from becoming fixed in the open ocean or from declining to zero in the southwestern portion of Long Island Sound.

in the mouse camouflage experiment, what is the independent variable? the dependent variable? explain.

The fur coat color of the mouse models is the independent variable because this is the variable that was changed intentionally by the researchers. Predation is the dependent variable, measured by the investigators and recorded as the proportion of the total number of attacked models.

WHAT IF? Suppose a molecular clock dates the divergence of two taxa at 80 million years ago, but new fossil evidence shows that the taxa diverged at least 120 million years ago. Explain how this could happen.

The gene (or genes) used for the molecular clock may have evolved more slowly in these two taxa than in the species used to calibrate the clock; as a result, the clock would underestimate the time at which the taxa diverged from each other.

MAKE CONNECTIONS Review Figures 14.6 and 14.12. Explain how a base-pair substitution that alters a coding region of the Adh locus could have no effect on amino acid sequence. Then explain how an insertion in an exon could have no effect on the protein produced.

The genetic code is redundant, meaning that more than one codon can specify the same amino acid. As a result, a substitution at a particular site in a coding region of the Adh gene might change the codon but not the translated amino acid, and thus not the resulting protein encoded by the gene. One way an insertion in an exon would not affect the gene produced is if it occurs in an untranslated region of the exon.

genome

The genetic material of an organism or virus; the complete complement of an organism's or virus's genes along with its noncoding nucleic acid sequences.

Does the graph indicate that gene flow is spreading fire-bellied toad alleles into the range of the yellow-bellied toad? Explain. (pg 450)

The graph suggests there has been gene flow of some fire-bellied toad alleles into the range of the yellow-bellied toad. Otherwise, all individuals located to the left of the hybrid zone portion of the graph would have allele frequencies close to 1.

How is the information encoded in DNA actually used by organisms?

The information in DNA is transcribed to mRNA and then translated into protein. The information in DNA is first transcribed to RNA and then translated into protein.

Why is the kingdom Monera no longer considered a valid taxon?

The kingdom Monera included bacteria and archaea, but we now know that these organisms are in separate domains. Kingdoms are subsets of domains, so a single kingdom (like Monera) that includes taxa from different domains is not valid.

INTERPRET THE DATA In which vertebrate lineage shown has the studied gene evolved most rapidly? Explain. I-----------------Drosophila(fly) --I I---------------Lancelet I--I I------------Zebrafish I--I I---------Frog I--I I------Chicken I--I I---Human I--I I---Mouse

The lineage of humans and mouse, since the length of the tree branches shows are representative of genetic change -- and humans and mice are largely differently developed species, then we can assume that their genes have developed most rapidly.

Birds and mammals have a four-chambered heart, but most reptiles have a three-chambered heart. How does this fact affect the construction of phylogenetic trees for these groups?

The most likely tree is not always the most parsimonious. Evidence shows that birds diverged from reptiles before mammals did; therefore, there is no common ancestor of birds and mammals that had a four-chambered heart. This character must have evolved twice.

Explain why "editing" is an appropriate metaphor for how natural selection acts on a population's heritable variation.

The naturally occurring heritable variation in a population is "edited" by natural selection because individuals with heritable traits better suited to the environment survive and reproduce more successfully than others. Over time, better-suited individuals persist and their percentage in the population increases, while less well-suited individuals become less prevalent—a type of population editing.

Homozygous C^YC^Y individuals cannot produce chlorophyll. The ability to photosynthesize becomes more critical as seedlings age and begin to exhaust the supply of food that was stored in the seed from which they emerged. Develop a hypothesis that explains the data for days 7 and 21. Based on this hypothesis, predict how the frequencies of the C^G and C^Y alleles will change beyond day 21.

The observed frequency of genotype CGCG is 47÷173 = 0.272.The observed frequency of genotype CGCY is 106÷173 = 0.613.The observed frequency of genotype CYCY is 20÷173 = 0.116.

(a) Calculate the observed frequencies of genotypes C^GC^G,C^GC^Y, and C^YC^Y at day 7. (The observed frequency of a genotype in a gene pool is the number of individuals with that genotype divided by the total number of individuals.) (b) Compare these frequencies to the expected frequencies calculated in question 2. Is the seedling population in Hardy-Weinberg equilibrium at day 7, or is evolution occurring? Explain your reasoning and identify which genotypes, if any, appear to be selected for or against.

The observed frequency of genotype CGCG is 49÷216 = 0.227.The observed frequency of genotype CGCY is 111÷216 = 0.514.The observed frequency of genotype CYCY is 56÷216 = 0.259.

What is true about an outgroup?

The outgroup should be less closely related to any members of the ingroup than those members are to each other. The outgroup should be known to be less closely related than any ingroup members are to each other based on other evidence such as paleontology, embryonic development, and gene sequences.

(a) Calculate the observed frequencies of genotypes C^GC^G,C^GC^Y, and C^YC^Y at day 21. (b) Compare these frequencies to the expected frequencies calculated in question 2 and to the observed frequencies at day 7. Is the seedling population in Hardy-Weinberg equilibrium at day 21, or is evolution occurring? Explain your reasoning and identify which genotypes, if any, appear to be selected for or against.

The population is in Hardy-Weinberg equilibrium.

The most parsimonious tree of evolutionary relationships can be inaccurate. How can this occur?

The principle of maximum parsimony states that the hypothesis about nature we investigate first should be the simplest explanation found to be consistent with the facts. Actual evolutionary relationships may differ from those inferred by parsimony owing to complicating factors such as convergent evolution.

gene expression

The process by which information encoded in DNA directs the synthesis of proteins or, in some cases, RNAs that are not translated into proteins and instead function as RNAs.

What is the relative fitness of a sterile mule? Explain.

The relative fitness of a mule is zero, because fitness includes reproductive contribution to the next generation, and a sterile mule cannot produce offspring.

branch point

The representation on a phylogenetic tree of the divergence of two or more taxa from a common ancestor. A branch point is usually shown as a dichotomy in which a branch representing the ancestral lineage splits (at the branch point) into two branches, one for each of the two descendant lineages.

biology

The scientific study of life.

biogeography

The scientific study of the past and present geographic distributions of species.

inquiry

The search for information and explanation, often focusing on specific questions.

artificial selection

The selective breeding of domesticated plants and animals to encourage the occurrence of desirable traits

evolutionary lineage

The sequence of ancestral organisms leading to a particular taxon; represented by a branch (line) in a phylogenetic tree.

MAKE CONNECTIONS In a region free of malaria, would individuals who are heterozygous for the sickle-cell allele be selected for or selected against? Explain.

The sickle cell allele will decrease in frequency. Under prolonged low oxygen conditions, some of the red blood cells of a heterozygote may sickle, leading to harmful effects. This does not occur in individuals with two normal hemoglobin alleles, suggesting that there may be selection against heterozygotes in malaria free regions. However, since heterozygotes are healthy under most conditions, selection against them is unlikely to be strong.

Hardy-Weinberg equilibrium

The state of a population in which frequencies of alleles and genotypes remain constant from generation to generation, provided that only Mendelian segregation and recombination of alleles are at work.

genomics

The study of whole sets of genes and their interactions within a species, as well as genome comparisons between species.

Pangaea

The supercontinent that formed near the end of the Paleozoic era, when plate movements brought all the landmasses of Earth together.

proteomics

The systematic study of the full protein sets (proteomes) encoded by genomes.

VISUAL SKILLS Consider a population in which heterozygotes at a certain locus have an extreme phenotype (such as being larger than homozygotes) that confers a selective advantage. Compare this description to the three types of selection depicted in Figure 21.13. Does this situation represent directional, disruptive, or stabilizing selection? Explain your answer.

The three modes of natural selection (directional, stabilizing, and disruptive) are defined in terms of the selective advantage of different phenotypes, not different genotypes. Thus, the type of selection represented by heterozygote advantage depends on the phenotype of the heterozygotes. In this question, because heterozygous individuals have a more extreme phenotype than either homozygote, heterozygote advantage represents directional selection

Speciation can occur rapidly between diverging populations, yet the time between speciation events is often more than a million years. Explain this apparent contradiction.

The time between speciation events includes the length of time that it takes for populations to begin diverging reproductively and the time it takes for speciation to be complete once this divergence begins. Speciation can occur rapidly, but it can take a long time for the population to diverge.

WHAT IF? Draw a phylogenetic tree that includes the relationships from Figure 20.16 and those shown here. Traditionally, all the taxa shown besides birds and mammals were classified as reptiles. Would a cladistic approach support that classification? Explain. I--Reptiles (including birds) --I I--Dimetrodon I---I I--Cynodonts I---I I--Mammals

The traditional classification provides a poor match to evolutionary history, thus violating the basic principle of cladistics—that classification should be based on common descent. Both birds and mammals originated from groups traditionally designated as reptiles, making reptiles (as traditionally delineated) a paraphyletic group. These problems can be addressed by removing Dimetrodon and cynodonts from the reptiles and by regarding birds as a group of reptiles (specifically, as a group of dinosaurs).

gene flow

The transfer of alleles from one population to another, resulting from the movement of fertile individuals or their gametes.

horizontal gene transfer

The transfer of genes from one genome to another through mechanisms such as transposable elements, plasmid exchange, viral activity, and perhaps fusions of different organisms.

VISUAL SKILLS Which of the trees shown here depicts an evolutionary history different from the other two? Explain. I-------------A ----I I--------B I----I I---C I----I I---D (a) I--------B I----I I---D I I----I ----I I---C I-------------A (b) I--------D I----I I---C I I----I ----I I---B I-------------A (c)

The tree in (c) shows a different pattern of evolutionary relationships. In (c), C and B are sister taxa, whereas C and D are sister taxa in (a) and (b).

Bioinformatics

The use of computers, software, and mathematical models to process and integrate biological information from large data sets.

WHAT IF? A locus that affects susceptibility to a degenerative brain disease has two alleles, V and v. In a population, 16 people have genotype VV, 92 have genotype Vv, and 12 have genotype vv. Is this population evolving? Explain.

There are 120 individuals in the population, so there are 240 al- leles. Of these, there are 124 V alleles—32 from the 16 VV individuals and 92 from the 92 Vv individuals. Thus, the frequency of the V allele is p = 124>240 = 0.52; hence, the frequency of the v allele is q = 0.48. Based on the Hardy-Weinberg equa- tion, if the population were not evolving, the frequency of genotype VV should be p2 = 0.52 * 0.52 = 0.27; the frequency of genotype Vv should be 2pq =2 0.52 0.48 = 0.5; and the frequency of genotype vv should be q2 = 0.48 *0.48 = 0.23. In a population of 120 individuals, these expected genotype frequen- cies lead us to predict that there would be 32 VV individuals (0.27 120), 60 Vv individuals (0.5 120), and 28 vv individuals (0.23 * 120). The actual numbers for the population (16 VV, 92 Vv, 12 vv) deviate from these expectations (fewer homo- zygotes and more heterozygotes than expected). This indicates that the population is not in Hardy-Weinberg equilibrium and hence may be evolving at this locus.

MAKE CONNECTIONS Review Concept 14.5 . Then explain how numerous base changes could occur in an organism's DNA yet have no effect on its survival and reproduction.

There are many portions of the genome that do not code for genes; mutations that alter the sequence of bases in such regions could accumulate through drift without affecting an organism's fitness. Even in coding regions of the genome, some mutations may not have a critical effect on genes or proteins.

How many genes must change in order to form a new species?

There is no set number of genes or loci that produces a new species. Genetic and environmental factors interact. Research has shown that genetic changes can be few or many. Other factors such as geographic isolation, catastrophic events, pollution, the establishment of hybrid zones, and so on, will affect speciation in different organisms.

What did Darwin observe about species on islands?

They are often closely related to species from the nearest mainland or neighboring islands. If a species that disperses from a mainland to an island succeeds in its new environment, it may give rise to several new species as populations spread to other islands in the archipelago.

How did Hutton's and Lyell's ideas influence Darwin's thinking about evolution?

They proposed that geologic events in the past were caused by the same processes operating today, at the same gradual rate. This suggested that Earth must be much older than a few thousand years. - they thought that geologic change occurs gradually, stimulating Darwin to reason that the slow accumulation of small changes could ultimately produce the profound changes documented in the fossil record - Darwin then thought that the only way there would have been time for evolution to occur was if earth was very old

(a) Predict what would happen if, after 22 months, guppies from the transplanted population were returned to the source pool. (b) Describe an experiment to test your prediction. What would most likely happen to those guppies? What data should you collect in order to test your prediction?

They would experience a higher rate of predation than the native population, and over time they would evolve and resemble the original source population predation rate on transplanted versus original source guppies, and color patterns of the population over time

Can you pick out the mossy leaf-tailed gecko lying against the tree trunk in this photo? How is the appearance of the gecko a benefit in terms of survival? Given what you learned about evolution, natural selection, and genetic information in this chapter, describe how the gecko's coloration might have evolved.

This coloration likely makes it easier for the gecko to hunt insects. This coloration likely makes it harder for the gecko to be seen by predators. 1. There are geckos of different coloration in a population. 2.Geckos that more closely resembled their background are less visible to predators and are more likely to survive, reproduce, and leave offspring. 3.The offspring would inherit the genes that generated the mossy bark coloration. 4.Over the generations, the coloration would become a closer and closer match to the tree bark.

MAKE CONNECTIONS After reviewing endosymbiont theory (see Figure 4.16), explain the specific positions of the mitochondrion and chloroplast lineages on this tree. (pg 415)

This tree indicates that the sequences of rRNA and other genes in mitochondria are most closely related to those of proteobacteria, while the sequences of chloroplast genes are most closely related to those of cyanobacteria. These gene sequence relationships are what would be predicted from endosymbiont theory, which posits that both mitochondria and chloroplasts originated as engulfed prokaryotic cells.

The time interval between speciation events is typically __________.

Times between speciation events vary widely and depend on generation times, causes of speciation, environmental factors, and chance. Speciation may occur on many different time scales. Remember that mutations or meiosis, both of which are largely governed by chance, must first provide some traits that can lead to reproductive isolation.

Explain how the types of data the researchers chose to collect enabled them to test their prediction.

Tracking the number and area of colored spots provided a quantitative way to compare the brightness of different populations.

Which of is an example of a postzygotic reproductive barrier?

Two fruit flies of different species produce sterile offspring. This is reduced hybrid fertility.

molecule

Two or more atoms held together by covalent bonds

WHAT IF? Predict whether a fossil of an extinct mammal that lived high in the Andes would more closely resemble present-day mammals that live in South American jungles or present-day mammals that live high in African mountains. Explain.

Unless convergent evolution occurred between andes mountain animals and african animals, this extinct animal would most likely resemble animals of present day SA jungles because it would have common ancestors since the Andes are within SA The fossil mammal species (or its an-cestors) would most likely have colonized the Andes from within South America, whereas ancestors of mammals currently found in African mountains would most likely have colonized those mountains from other parts of Africa. As a result, the Andes fossil species would share a more recent common ancestor with South American mammals than with mammals in Africa. Thus, for many of its traits, the fossil mammal species would probably more closely resemble mammals that live in South American jungles than mammals that live on African mountains. It is also possible, however, that the fossil mammal species could resemble the Af- rican mountain mammals by convergent evolution (even though they were only distantly related to one another).

SYNTHESIZE YOUR KNOWLEDGE This honeypot ant (genus Myrmecocystus) can store liquid food inside its expandable abdomen. Consider other ants you are familiar with, and explain how a honeypot ant exemplifies three key features of life: adaptation, unity, and diversity.

When the environment becomes too hostile to aid the search for food (that is, dry season), the honeypot ant which can be referred to as repletes shows adaptation to its environment with the intelligent ability to store liquid food inside its abdomen. Their expandable abdomen serve as "living larders" for their entire colony because they are storage vessels. It exemplifies unity because during a dry season, nutrients become more scarce. Very few flowers blossoms, and fewer insects will approach the colony which means less access to food. Honeypot ants turn to their living larders, which would have swollen to the size of grapes. When the colony is in need of nutrient-rich liquids, a worker ant will stroke a replete's antennae. This signals the replete to regurgitate the liquid. A worker can eat the liquid itself, or carry it to another member of the colony. It exemplifies diversity because it is different from other ants in its ability to store liquid food in its expandable abdomen. Many insects store sweet liquid for later use. For example, honeybees usually stores liquid in their combs. Honeypot ants, are the only insects to store the liquid in their own bodies.

Which is the best example of gene flow?

Wind blows pollen from one population of plants to another and cross-fertilization occurs. Over several generations, the two populations will become more similar.

MAKE CONNECTIONS Scientific hypotheses must be testable (see Concept 1.3 ). Applying this criterion, are Cuvier's explanation of the fossil record and Lamarck's hypothesis of evolution scientific? Explain your answer in each case.

Yes, By this criterion, Cuvier's explanation of the fossil record and Lamarck's hypothesis of evolution are both scientific. Cuvier thought that species did not evolve over time. He also suggested that sudden, catastrophic events caused extinctions in particular areas and that such regions were later repopulated by a different set of species that immigrated from other areas. These assertions can be tested against the fossil record, and his assertion that species do not evolve has been demonstrated to be incorrect. With respect to Lamarck, his principle of use and disuse can be used to make testable predictions for fossils of groups such as whale ancestors as they adapted to a new habitat. Lamarck's principle of use and disuse and his associated principle of the inheritance of acquired characteristics can also be tested directly in living organisms; these principles have been shown to be incorrect.

Should an experiment test only one variable at a time? Why or why not?

Yes, an experiment should only test one variable at a time. This ensures that the experimental outcome is clearly due to one identifiable factor. Treatment of an experimental group should differ from treatment of a control group by only a single variable so that the researcher can determine what causes the observed effect.

Can factors that cause sympatric speciation also cause allopatric speciation? Explain.

Yes. Sympatric speciation can be promoted by factors such as polyploidy, habitat shifts, and sexual selection, all of which can reduce gene flow between the subpopulations of a larger population. But such factors can also occur in allopatric populations and hence can also promote allopatric speciation.

Do these results indicate that hybridization can lead to the transfer of adaptive alleles? Explain.

Yes; the r allele was not present in A. coluzzii before 2006, but was present in A. coluzzii populations once hybrid individuals carrying the r allele were observed in 2006.

DRAW IT Which branch point represents the most recent common ancestor of cetaceans and seals. Explain why that ancestor would not be part of a cetacean-seal group defined by their similar body forms. I---Other even-toed ungulates I---I I---Hippopotamuses I I---I I I---Cetacean -I I I---Seals I I---I I---I I---Bears I---Other carnivores

You should circle the branch point that is drawn farthest to the left (the common ancestor of all taxa shown). Both cetaceans and seals descended from terrestrial lineages of mammals, indicating that the cetacean-seal common ancestor lacked a streamlined body form and hence would not be part of the cetacean-seal group

DRAW IT With rough sketches, draw a biological hierarchy similar to the one in Figure 1.3 but using a coral reef as the ecosystem, a fish as the organism, its stomach as the organ, and DNA as the molecule.

Your figure should show the following; (1) for the biosphere, Earth with an arrow coming out of a tropical ocean; (2) for the ecosystem, a distant view of a coral reef; (3) for the community, a collection of reef animals and algae, with corals, fishes, some seaweed and any other organisms you can think of; (4) for the population, a group of fish of the same species; (5) for the organism, one fish from your population; (6) for the organ, the fish's stomach, and for the organ system, the whole digestive trace (see Chapter 33 for help); (7) for a tissue, a group of similar cells from the stomach; (8) for a cell, one cell from the tissue, showing its nucleus and a few other organelles, (9) for the organelle, the nucleus, where most of the cell's DNA is located; and (10) for a molecule, a DNA double helix. Your sketches can be very rough.

A number of mosquito populations today are resistant to insecticides that were once quite effective. Biologists think that insecticide resistance evolved in mosquitoes because __________.

a few mosquitoes were probably resistant to the insecticide before it was ever used, and these individuals were more likely to survive and reproduce The degree of adaptation can be extended only within the realm of genetic variability present in the population.

species

a population or group of populations whose members can interbreed and produce fertile offspring, but do not produce viable, fertile offspring with members of other such groups.

sexual selection

a process in which individuals with certain inherited characteristics are more likely than other individuals of the same sex to obtain mates

Explain the role of gene flow in the biological species concept.

a species is a group of populations whose members interbreed and produce viable, fertile offspring; thus, gene flow occur. In contrast, members of different species don't interbreed, so there is no gene flow between their populations. Overall in the biological species concept, species can be viewed as designated by the absence of gene flow.

hypothesis

a testable explanation for a set of observations based on the available data and guided by inductive reasoning. A hypothesis is narrower in scope than a theory.

A theory is _____.

a well-supported concept that has broad explanatory power Theories are comprehensive and become widely accepted if they are supported by a large body of evidence.

Approximately 1 out of every 2,500 Caucasians in the United States is born with the recessive disease cystic fibrosis. According to the Hardy-Weinberg equilibrium, approximately what percentage of people are carriers?

about 4% If q2 equals 1/2,500, then q equals .02, p equals 0.98, and 2pq equals approximately 0.04, or 4%.

Which is outside the realm of scientific inquiry? - explaining naturally occurring events - determining the efficacy of a low-carbohydrate diet - addressing ethical dilemmas - formulating testable hypotheses in seeking natural causes for natural phenomena - determining the physical causes for physical phenomena

addressing ethical dilemmas Questions of ethics are not amenable to solution by experiments or observations.

All known organisms transcribe genetic information to protein molecules via the same genetic code. This finding strongly supports the hypothesis that __________.

all organisms are descended from a single common ancestor A common genetic code is overwhelming evidence that all life is related.

Summarize evidence that the yup flower-color locus acts as a prezygotic barrier to reproduction in two species of monkey flowers. Do these results demonstrate that the yup locus alone controls barriers to reproduction between these species? Explain.

alleles at yup locus can influence pollinator choice, which provides primary barrier to interspecific mating. Nevertheless, experiment does not prove that yup locus alone controls barriers to reproduction between M. lewisii and M. cardinalis; other genes might enhance effect of yup locus or cause entirely different barriers to reproduction

The Galápagos finches collected by Darwin are thought to have descended from a very small parent population. Different species of finches that are confined to only one or two islands most likely arose by __________.

allopatric speciation As the finches spread out to different islands and became isolated from each other, gene flow between them ceased, and new species arose.

Experimentation is only one part of the process of scientific inquiry, but it is a very important step because it __________.

allows rejection of a hypotheses Experimentation is important because hypotheses can be rejected when contradictory evidence emerges.

science

an approach to understanding the natural world

The wings of a hummingbird and a bee are __________.

analogous Structures that are similar as a result of convergent evolution are referred to as analogous structures.

Which would be the least useful in determining the relationships among various species?

analogous structures Analogous structures are the result of convergent evolution and may not reflect the relationships among the species that share them.

Emergent properties of living systems are defined as properties that __________.

are due to the arrangement and interactions of parts as complexity increases Emergent properties are novel properties not present at the level of biological organization just below. Emergent properties arise from the arrangement and interactions of parts as complexity increases.

The breeding of plants and animals for particular traits by humans is called __________.

artificial selection This is the mating of organisms with desirable characteristics. One example is the breeding of cows to produce offspring with higher milk yields.

Which type of selection maintains stable frequencies for two or more phenotypic forms in a population?

balancing selection Balancing selection preserves variation through many mechanisms, such as heterozygote advantage and frequency-dependent selection.

WHAT IF? Fossils show that dinosaurs originated between 200 and 250 million years ago. Would you expect the geographic distribution of early dinosaur fossils to be broad (on many continents) or narrow (on one or a few continents only)? Explain.

because the earth's landscape was Pangea at this time, and dinos were large and mobile, it is likely they lived on many different continents but when Pangea broke apart their fossils would have moved with the rocks they were deposited in

In Figure 22.14, Efforts are underway to develop drugs that target S. aureus specifically and to develop drugs that slow the growth of MRSA but do not kill it. Based on how natural selection works and on the fact that bacterial species can exchange genes, explain why each of these strategies might be effective.

both strategies should increase the time it takes S. aureus to become resistant to a new drug. if a drug that harms S. aureus does not harm other bacteria, natural selection will not favor resistance to that drug in the other species. this would decrease the chance that S. aureus would acquire resistance genes from other bacteria- thus slowing the evolution of resistance. similarly, selection for resistance to a drug that slows the growth but does not kill S. aureus is much weaker than selection for resistance to a drug that kills S. aureus- again slowing the evolution of resistance

An earthquake hits a small island. All but a small group of closely related lizards are eliminated, and the survivors spread out over the island. This is an instance of __________.

bottleneck effect This is a description of the bottleneck effect.

In practice, how do scientists distinguish most species?

by using the morphological species concept This concept is used to characterize a species in terms of body shape, size, and other distinct structural features.

SCIENTIFIC INQUIRY DRAW IT (a) Draw a phylogenetic tree based on characters 1-5 in the table below. Place hatch marks on the tree to indicate the origins of characters 1-6. (b) Assume that tuna and dolphins are sister species, and redraw the phylogenetic tree accordingly. Use hatch marks to indicate the origins of characters 1-6. (c) Determine how many evolutionary changes are required in each tree. Identify the most parsimonious tree. Character / Lancelet / Lamprey / Tuna / Salamander / Turtle / Leopard / Dolphin / Backbone / 0 / 1 / 1 / 1 / 1 / 1 / 1 / Hinged jaw / 0 / 0 / 1 / 1 / 1 / 1 / 1 / Four limbs / 0 / 0 / 0 / 1 / 1 / 1 / 1* / Amnion / 0 / 0 / 0 / 0 / 1 / 1 / 1 / Milk / 0 / 0 / 0 / 0 / 0 / 1 / 1 / Dorsal Fin / 0 / 0 / 1 / 0 / 0 / 0 / 1 (*Although adult dolphins have only two obvious limbs (their flippers), as embryos they have two hind-limb buds, for a total of four limbs.)

c) the tree in a requires 7 evolutionary changes, while the tree in b requires 9 evolutionary changes. Thus, the tree in a is more parsimonious, since it requires fewer evolutionary changes.

Which is the fundamental unit of structure and function in living organisms?

cell The cell is the fundamental structural and functional unit of both single-celled and multicellular organisms.

Which gives the correct sequence of biological organization, from simplest to most complex?

cell, tissue, organ, organisms, population, community (ordered in increasing complexity)

Evidence from molecular biology supports the theory of evolution by demonstrating that __________.

closely related organisms have more similar DNA and proteins Evolutionary relationships among species are reflected in their DNA and proteins. If two species have similar libraries of genes and proteins, with sequences of monomers that match closely, the sequences most likely descended from a common ancestor.

The phylogenetic species concept emphasizes __________.

common ancestry The phylogenetic species concept emphasizes common ancestry.

Which method for establishing phylogenetic relationships among organisms has been developed most recently?

comparing the component sequences of proteins and nucleic acids The comparison of nucleic acids and proteins has become a powerful addition to the other comparative methods that systematists use to measure evolutionary relationships among species.

In an ecosystem, nutrients __________ and energy __________.

cycle, flows through Nutrients are always recycled. Energy flows through the system, typically entering as solar energy and leaving as heat.

With evolution as the core theme of biology, we can explain traits shared by organisms as evidence of __________ and traits that differ among organisms as evidence of __________.

descent from a common ancestor, adaptation through natural selection Many shared traits have been derived from common ancestry. The modifications to those shared traits can usually be attributed to adaptive evolution by natural selection.

Evolution is broadly defined by Darwin as __________.

descent with modification Darwin used that phrase in proposing that Earth's many species are descendants of ancestral species that differed from present-day species.

sexual dimorphism

differences between the secondary sex characteristics of males and females of the same species

Antibiotic resistance in bacteria is an example of what?

directional selection Directional selection favors those individuals at one end of the spectrum that best adapt to a new situation—in this case, the presence of an antibiotic.

A population of squirrels is preyed on by small hawks. The smaller squirrels can escape into burrows. The larger squirrels can fight off the hawks. After several generations, the squirrels in the area tend to be very small or very large. What process is responsible for this outcome?

disruptive selection Disruptive selection favors individuals at both ends of the phenotypic spectrum.

An important challenge to traditional, pre-Darwinian ideas about species was the observation that seemingly dissimilar organisms such as hummingbirds, humans, and whales have similar skeletal structures. This most directly suggested to biologists that __________.

dissimilar organisms might have evolved from a distant, common ancestor Similarity in characteristics resulting from a common ancestor is known as homology, and such anatomical signs of evolution are known as homologous structures.

A phylogenetic tree of bird families constructed by cladistic analysis would be a hypothesis about what?

evolutionary relationships among bird families Phylogenetic trees are constructed to reflect the evolutionary relationships among organisms.

Animals that possess homologous structures probably __________.

evolved from the same ancestor "Homology" refers to similarity attributable to common ancestry.

Stabilizing selection __________.

favors intermediate variants in a population Stabilizing selection culls extreme variants from the population.

Which type of mutation plays the most important role in increasing the number of genes in the gene pool?

gene duplication Gene duplication is an important source of new genetic material.

Which evolutionary mechanism does not contribute to the process of allopatric speciation?

gene flow Gene flow between populations tends to homogenize the populations, reducing the likelihood of speciation occurring.

A population of 15 birds inhabits a fairly new island. Ten of the birds are dark brown and five of them are light brown. By chance, two of the dark brown birds and three of the light brown birds die before producing any offspring. All of the birds in the next generation are dark brown. This change in phenotypic frequency can be attributed to __________.

genetic drift In small populations, random chance can greatly affect allele frequencies.

neutral variation

genetic variation that does not provide a selective advantage or disadvantage.

The binomial system assigns to each organism a unique name that describes its __________.

genus and species Each species is assigned a two-part Latin name, or binomial. The first word is the genus and the second word is the species. Species that are very similar are grouped into the same genus.

The best method for determining whether bean plants require sodium is to __________.

grow bean plants with and without sodium In hypothesis-based scientific inquiry, the experiment must have an experimental group and a control group. In this case, growing bean plants with sodium would constitute the experimental group, and growing bean plants without the addition of sodium would constitute the control group.

At the time Darwin voyaged on the survey ship HMS Beagle, the popularly accepted theory in Western culture that explained the origin of Earth's plants and animals held that the various species __________.

had been created by divine intervention a few thousand years before Darwin's view of life contrasted sharply with the conventional paradigm of an Earth only a few thousand years old, populated by unchanging forms of life that had been individually made during a single week in which the Creator formed the entire universe.

Summarize the different lines of evidence supporting the hypothesis that cetaceans descended from land mammals and are closely related to even-toed ungulates.

hind limbs, ankle bone, and DNA sequence

Which most accurately measures an organism's fitness?

how many fertile offspring it produces That is the definition of fitness.

SCIENTIFIC INQUIRY Scientists search the scientific literature by means of electronic databases such as PubMed, a free online database maintained by the National Center for Biotechnology Information. Use PubMed to find the abstract of a scientific article that Hopi Hoekstra published in 2016 or later.

https://pubmed.ncbi.nlm.nih.gov/27496333/

Which describes an example of neutral variation?

human fingerprints As far as we know, this variation from human to human is neutral; that is, it has no selective advantage.

In a particular pea population, suppose that flowers with the white phenotype are favored by NS. What would happen over time to the frequency of the p allele in the population and why?

if NS continues to favor white phenotype then the frequency of white individuals will increase because the recessive allele's appearance will increase

A new species can arise in a single generation __________.

if a change in chromosome number creates a reproductive barrier This most often happens in plants.

Which of the following would help strengthen the researchers' conclusion?

if no other characteristics differed significantly between the source and transplanted populations

At which point is a scientific investigator most likely to use deductive reasoning? - after the careful analysis of both the qualitative and the quantitative data recorded in the study - in establishing a test of a hypothesis - in rephrasing an alternative hypothesis - during the formulation of a hypothesis - during initial observation(s)

in establishing a test of a hypothesis In deduction, reasoning flows from the general to the specific. Deductive testing would take the form of "if...then" logic.

A taxon __________.

is a formal grouping at any given level A taxon is a formal grouping of organisms at any given level, such as species, genus, or phylum.

In experimental procedures, repetition of the procedures __________.

is necessary before concluding that a given set of results is correct Repetition of tests is crucial to the process of science. Only when a result is repeatable can it be considered correct.

To describe evolution as a "scientific theory" means that __________.

it is a broad model that is supported by many observations and much experimental evidence Evidence from many fields of biology makes evolution the "central, unifying principle of biology."

DRAW IT The bear family (Ursidae) is more closely related to the badger/otter family (Mustelidae) than to the dog family (Canidae). Use this information to redraw Figure 20.4.

leopard bear American badger European otter coyote cray wolf

When they were first sold, aerosol insecticides were highly effective in killing flies and mosquitoes. Now, several decades later, a much smaller proportion of these insects from a given species die when sprayed. The reason fewer insects die when they are sprayed is that __________.

many mosquitoes today are descendants of mosquitoes with insecticide-resistant characteristics Individuals whose characteristics best fit them to their environment are likely to leave more offspring than are less fit individuals. This unequal ability of individuals to survive and reproduce will lead to a gradual change in a population, with favorable characteristics accumulating over the generations.

Sometimes two phenotypically different populations interbreed to a limited extent, so that it is difficult to determine whether they are clearly separate species. This is not a concern to scientists because this __________.

may indicate that the formation of a new species is in progress This could indicate that although a reproductive barrier is becoming established, it is not yet complete.

Which reproductive barrier actually prevents individuals of closely related species from copulating successfully?

mechanical isolation With mechanical isolation, mating cannot occur successfully due to incompatibility of reproductive parts.

Sexual recombination includes the shuffling of chromosomes in __________ and fertilization.

meiosis I Sexual recombination occurs with the crossing over and independent assortment of chromosomes, both of which occur during meiosis.

Which can form entirely new alleles?

mutations Although mutations most often do not result in favorable adaptations, they are the primary way that new alleles come about.

MAKE CONNECTIONS Review Figure 19.14. Which mode of selection has occurred in soapberry bug populations that feed on the introduced goldenrain tree? Explain. (pg 432)

natural selection The length of each bug's beak gradually became shorter over time. Bugs developed short beaks, because they needed them to feed on the small fruits. Bugs with shorter beaks had more access to food, allowing them to produce more offspring.

In Darwin's view of descent with modification __________.

natural selection can improve the match between an organism and its environment Natural selection acting in a particular environment can increase the compatibility between an organism and its environment.

Is speciation something that happened only in the distant past, or are new species continuing to arise today? Explain.

new species can begin to form whenever gene flow is reduced between populations of the parent species. Reductions can occur in many ways: A new, geographically isolated population may be founded by a few colonists; new habitat; or sexual selection.

Which organism is most likely to be subject to allopatric speciation?

pine trees in Alaska and pine trees on the island of Madagascar This is geographic isolation that could lead to allopatric speciation, for it is unlikely that pollen from one locale could get to the other locale.

The smallest unit that can evolve is a(n) __________.

population Evolution can be measured only as changes in relative proportions of heritable variations in a population over many generations.

Prezygotic barriers __________.

prevent fertilization of gametes from members of closely related species These barriers hinder the fertilization of ova if members of sibling species attempt to mate.

Three species of frogs, Rana pipiens, Rana clamitans, and Rana sylvatica, all mate in the same ponds, but they pair off correctly because they have different calls. This is a specific example of a __________ barrier, called __________.

prezygotic, behavioral isolation It occurs before mating and is due to different behaviors.

According to the theory of evolution, anatomical and molecular homologies should __________.

produce similar patterns of evolutionary relatedness According to evolutionary theory, anatomical and molecular homologies should generally produce evolutionary patterns that corroborate each other.

Which set of conditions is required for Hardy-Weinberg equilibrium?

random mating, no natural selection, and a large population These three conditions, along with no mutations and no movement of alleles into or out of the population, are required for Hardy-Weinberg equilibrium.

Sequencing a protein to discover the order of the amino acids that make it up is an example of __________. Understanding how that protein works with other proteins in a cell is an example of __________.

reductionism, systems biology Reductionism seeks to reduce complex systems to simpler, more manageable components. System biology seeks to understand how all the parts function together.

Vestigial organs are __________.

remnants of structures that were useful to an organism's ancestors For example, some species of snakes have remnants of pelvic and leg bones.

Unlike a regular phylogenetic tree, phylogenetic trees with branch lengths proportional to time can be used to __________.

represent the chronological time that has passed since two groups diverged from a common ancestor In this type of tree, called an ultrametric tree, the branching pattern is the same as in a normal phylogenetic tree, but all the branches that can be traced from the common ancestor to the present are of equal length.

Which condition is necessary for speciation to occur?

reproductive isolation The biological species concept hinges on reproductive isolation.

At the time Charles Darwin sailed on the HMS Beagle, __________.

several biologists had proposed that species might change over time, but none had suggested a convincing mechanism that might cause the change By the end of the 18th century, several naturalists had suggested that life evolved along with the evolution of Earth. An important facet of Darwin's work is the concept of natural selection as the cause of adaptive evolution.

In the case of the Lake Victoria cichlids, sympatric speciation has been shown to be driven, at least in part, by __________.

sexual selection Experiments have shown that females select mates on the basis of coloration. This behavioral isolation may have been a major factor in the speciation of cichlids.

Selection that acts over evolutionary time to preserve traits that increase an individual's ability to mate is known as __________.

sexual selection In sexual selection, secondary sexual characteristics that may not seem to be adaptive in the general sense may give an individual an advantage in gaining a mate.

When gene flow between two populations ceases, the potential for __________ exists.

speciation Reproductive isolation (the cessation of gene flow) is necessary for speciation.

Consider three different species: A, B, & C. Species A and B are in the same phylum. Species A and C are in the same order. From this information you can conclude that __________.

species C could be in the same class as species A and B If species A and species C are in the same order, then they must also be in the same class. Because you know that species A and species B are in the same phylum, it is possible that they are in the same class, but different orders.

Birds with average-size wings survived a severe storm more successfully than other birds in the same population with longer or shorter wings. If severe storms occur regularly, then over time, one should expect these storms to bring about __________.

stabilizing selection Stabilizing selection favors the intermediate trait.

For several years, scientists have warned doctors of the danger of overprescribing antibiotics such as penicillin. Scientists are concerned because __________.

strains of microorganisms that are resistant to these drugs will be selected for A beneficial mutation can increase in frequency in a bacterial population very rapidly. If a single individual in the population harbors a mutation that renders it resistant to an antibiotic, there may be millions of resistant bacteria within several hours of antibiotic treatment.

SYNTHESIZE YOUR KNOWLEDGE Suppose that females of one population of strawberry poison dart frogs (Dendrobates pumilio) prefer to mate with males that are orange-red in color. In a different population, females prefer males with yellow skin. Explain how such differences could arise and how they could affect the evolution of reproductive isolation in allopatric versus sympatric populations.

strawberry poison dart frogs have different color and pattern between different populations. Such differences could be due to allopatric speciation. These populations would have been isolated from each other for thousands of years. These phenotypic changes would have been induced because of geographically isolation. This would have used genetic drift to achieve such color diversity. The population would have adapted to the new environment and new mutations would have selected gene mutations, natural selection and genetic drift are the mainly involved in the allopatric speciation. However, such color patterns are generally related to sexual selection too

Lyell's principle of uniformitarianism __________.

strongly influenced Darwin's view of how living organisms could change over time Darwin eventually embraced the idea of an old Earth and rejected the idea that living organisms had been specially created. He applied the idea of slow change to the evolution of organisms.

Bird guides once listed the myrtle warbler and Audubon's warbler as distinct species that lived side by side in parts of their ranges. However, recent books show them as eastern and western forms of a single species, the yellow-rumped warbler. Apparently, the myrtle warbler and Audubon's warbler __________.

successfully interbreed and produce fertile offspring That is the definition of biological species.

A discipline that is focused on classifying organisms and determining their evolutionary relationships is __________.

systematics Systematists study morphological and behavioral resemblances as well as molecular similarities and differences to devise theories about relationships.

There are two groups of pine trees that appear to be very similar phenotypically and genotypically. However, one releases pollen in January, when the female structures of that group are receptive, and one in March. What kind of reproductive barrier is this?

temporal isolation Temporal isolation occurs when two similar species have sexual readiness at different times.

Two species of water lilies in the same pond do not interbreed because one blooms at night and the other during the day. The reproductive barrier between them is an example of __________.

temporal isolation The gametes of species that breed during different times of the day, different seasons, or different years do not have an opportunity to combine and create a zygote.

A company was testing a new drug it thought would help decrease the risk of transmission of viruses from mother to fetus. In an experiment to test the compound, an investigator gave 400 pregnant female rats a small dose of the experimental drug and inoculated each with a type of virus known to cause disease in rats. At the same time, 400 other pregnant rats were given only the virus. Of the rat pups born to the females that received both the virus and the drug, 203 showed no symptoms of the disease; 205 rat pups born to the virus-only females showed symptoms. From these data, we can best conclude __________.

that the drug seems to have little effect on viral transmission at the dosage given Approximately equal numbers of pups were affected in both groups (experimental group 400 − 203 = 197, control group = 205), suggesting that the drug was probably not effective at the dosage given.

Carolus Linnaeus is considered to be the founder of __________, and he __________.

the binomial classification system, thought that resemblances among different species reflected the pattern of their creation Although he is considered the founder of taxonomy, Linnaeus accepted the theory of creation.

What is the focus of the branch of biology called taxonomy?

the classification of life-forms by their similarities and differences This is exactly what the study of taxonomy seeks to accomplish.

Using cladistic analysis, a taxonomist wishes to construct a phylogenetic tree showing the relationships among various species of mammals. Which would be least useful for this purpose? - DNA base sequences - the fact that teeth vary among types of mammals - data about skull bones - the fact that all mammals have hair - descriptions of various types of limbs (wings, legs, flippers, etc.)

the fact that all mammals have hair As a shared derived character, hair cannot be used to determine the phylogeny of mammals.

In a large population of randomly breeding organisms, the frequency of a recessive allele is initially 0.3. There is no migration and no selection. Humans enter this ecosystem and selectively hunt individuals showing the dominant trait. When the gene frequency is reexamined at the end of the year, __________.

the frequency of the homozygous dominants will go down, the frequency of the heterozygous genotype will go down, and the frequency of the homozygous recessives will go up The frequency of individuals who express the dominant phenotype (homozygous dominants and heterozygotes) should decline, and the proportion of individuals who express the recessive phenotype (homozygous recessives) should increase.

Predict how the transfer of the r allele to A. coluzzii populations could affect the number of malaria cases in the years immediately following the transfer.

the number of malaria cases will increase

Paleontology

the scientific study of fossils.

The energy used by most organisms for metabolism and growth ultimately comes from __________.

the sun Energy for most life ultimately comes from the light energy of the sun, trapped in the chemical bonds of molecules synthesized by photosynthetic organisms.

According to the punctuated equilibrium model of evolution, __________.

the tempo of evolution consists of abrupt episodes of speciation among long periods of equilibrium This is the exact description of the punctuated equilibrium model of evolution.

In Figure 22.13, when soapberry bug eggs from a population fed on balloon vine fruits were reared on goldenrain tree fruits (or vice versa), the beak lengths of the adult insects matched those in the population from which the eggs were obtained. Interpret these results.

these results show that being reared from the egg stage on one plant species or the other did not result in the adult having a beak length appropriate for that host; instead, adult beak lengths were determined primarily by the population from which the eggs were obtained. because an egg from a balloon vine population likely had long-beaked parents, while an egg from a goldenrain tree population likely had short-beaked parents, these results indicate that beak length is an inherited trait

State the researchers' hypothesis, and identify the independent and dependent variables in this study. Explain why the researchers used four mating combinations for each pair of populations.

to compare the proportion of successful matings within populations to the proportion of successful matings between populations

Radon is a radioactive gas that seeps into homes from the soil. It is thought to be a cause of lung cancer. A research team gathers large amounts of data on basement radon concentrations and lung cancer rates and conclude that the more radon there is in a home, the more likely that people living in those homes will develop lung cancer. After the study is published, other researchers criticize it by asserting that the studied neighborhoods with higher radon concentrations also have a higher percentage of older people and a higher percentage of cigarette smokers than the low-radon neighborhoods. Both advanced age and cigarette smoking increase the risk of lung cancer. This criticism, if correct, shows that the radon study suffered from __________.

uncontrolled variables An important feature of the scientific process is the controlled experiment. In a controlled experiment, two groups are established and treated exactly alike, except for the one variable the experiment is designed to test.

SCIENTIFIC INQUIRY Based on the results of the mouse coloration case study, propose a hypothesis researchers might use to further study the role of predators in the natural selection process.

you might wonder what would happen if a population of beach mice lived in an rea where predators were absent


Ensembles d'études connexes

The Lipids: Triglycerides, Phospholipids, & Sterols

View Set

European History Through Art TEST

View Set

Liver, Bile, Gallbladder, Pancreas

View Set

Social Statistics for exam 2 chapters

View Set

Marketing test 2 sample questions (ch 4, 5

View Set

SAChE Module hazards and risk AJJ

View Set